2.OnExamination MRCP 1 Gastroenterology 2020 Q.bank

You might also like

Download as pdf
Download as pdf
You are on page 1of 348
A31-years-old male teacher attends clinic with his partner who tells you thathe has memory problems. The only other symptom is intermittent diarthoea over the preceding four months. He has limited vertical eye movements and exhibits rhythmic simultaneous eye and mouth movements. Which pathogen is most likely to be the cause of his symptoms? 2 select 1 option) © Clostridium botulinum OHV © Prion protein © Salmonella enteritidis © Tropherymawhippleit oe) a) ‘ganation ‘sso tousnauton ‘Thesgecen ret neste ha pis danse ction peyme ee Noprsaeoga mnfesatens ct ipl’ dase te mee cammorancnuce conic dare mabupowih Neclpatesatersinane ari pegesine oanmertthger mention inasecaen wae, clos atom lena aaa unt nen EES ‘Tredgron «mae ssc ohn oat denesaning betas wthn emucsnon PAS create. cunultn ef enteral AS se macepase ine amne Pe, alee net patience An Sete ener posible but term Acalmeemens welcte un Crate ot enn CD els pode ete athe mses ean moe ofan cop monies neearreineconsnte Shamim ie ys ei . ow i “ woot “ so nin apt) ES Fee ‘Autoimmune hepattis Hepatitis te ID No Hed enphoma ee -) Explanation ‘Theansweris Hepatitis C The history of previous drug abuse. with ised LFTs, shoul lead to consider viralhepatits asa cause ‘other symptoms, andi is HESABHEE rich fs most commonly associated with cyoglobuinaemia Antivials*/-Bcell {targeting eaens such rita ore the mainstay ontervention ‘Cyoglobuln ar also deserbein association with both BUGTAUREESAI and primary any GB. although both are less itely diagnoses given the previous history fIV rug abuse Although there are some smallpaipable lymph nodes. mare ‘masked changes including potently enlarged hier lymph nodes onthe CAR Wouldbe expectedin potiets with nom Hocsin'siyraphoma, Which of the following gut hormones stimulates acid secretion in the stomach? (Please select 1 option) Q Cholecystokinin (eck) © Gastnn OQ Polypeptide P © Secretin Q Vasoactive intestinal peptide (VIP) ae eK. « Cholecystokinin (CCK) O° 6th am, © paneer [© Seren © Vasoactive nts ponte ve) EERE Explanation Gastrn is secreted from the antrum of the stomach when stimulated by parasympathetic nerves and the presence of amino acids inthe stomach Ie stimulates gastric motility, growth and acid secretion, and intestinal motility ‘The secration of gastrin fs inhibited by acid in stomach and somatostatin 'A20-yearold man was found to hae ron deficiency anaemia when he weno donate bloc. The Blood Transfusion Senvice ‘contacted his general practtanes who refered the patient tome outpatient cic frturtherIwestgaton ‘General, the patent was very well He nada good appette is eight wassteady and ne ate 3 normal det Henada normal bowel habit anc had neve passed any blood. mucus or athoea in his stoals. The patent denied knowedge of any overt ood os fom any ater source, is nel practioner had organised an open access endoscopy which was normal duodenal biopsies wer unremarkable. enacts bite kno of ie fmiy stony ae meter ac las inchlcmirn and ae arantne wat anny chia Hie {ather hadi of what he thought was secondary ler andlung cancer bute was unsure ‘On general physi examination he was t and athletic. The skin and mucosal membranes wereunremarkabe Pulse was 70 beats per minut ard regular with blod peseur of 132/78 mmlig. Hert sounde ware normal and the chest was clear. ie _iedomen was soft and nr tender with ne palpablemasses or organs, Rectal examination was norman viewing the rectal ues though arigasgmoicoscope the colonic mucoss was covered i innumerable pops. at specie genetic abnormality responsible ror this appearance? © Germine mutation ofthe STAI gene. chramoseme 19 (0 Homezygous mutation ofthe rH gene © Lossof the APC ene on chromosomes ‘© tations in msmaten rear genes (eg MSH2) (© mutation oftnep53 tumour suppressor gene Comminemutaion tthe STR anon chrome 19 mazjgousmuaton rine Mi gene Losotventeneon tenosenes CEERI satonsin mamaten cpa gees og MSD) © Msinetestemr se oe SITY planation 1 Rian Sp GSC apis FAP, casedy eles fhe AC genean thelorgam afehamoreme'S Eeuzlshersdtame isan tos dominate core cause amine arationof STI err {opie tise Tees val etd entbalongamio vomesone 19 Pei letanr nrc imessnanartonseuspayps isu S09 asoates we ea ementn eregtay nenpolgoss elon cancer OANFEE ANCHE polyooes ot cause mR flpsassuepesteintiscase In rc aectes avousinert 3 mutzton mene ct se genes HvONeCInONA MISE par cua MSH2, ML. andPS2 Homozrspusmuatens ithe MH ane have beenasocted wth apnentpecf multe colret sdeneas whit cancer Thsaceuns fra poporenct FAP patents wanes rahegenc APC mUStON. msatensin ne ps tumour suppressor geese found many eer cancers While mutatonspS3 seein cases ot elon cance, the guestn aso he specie mutton acca win paypal Themain steel inti questonistenvean FAP and IS but th akc prc pigmentation cure FHP athe capns 39-year-old sailor has had repeated visits to his GP for heartburn and dyspepsia. He had been an irregular visitor and his treatments had been discontinuousiin the past. This time, after an acute episode of dyspepsia and abdominal pain, he underwent an upper Gi endoscopy which showed Barret's esophagus. Histopathology showed very low-grade dysplasia. Whats the bast next line of management? Pleas: 1 option) © Attialof PPI therepy © Endoscopic ablation therapy © Endoscopic mucosal resection © Lower esophageal resection © Onlyobservation ee ee Aide) TT Endoscopic ablation therapy Endoscopic mucosal resection © Loweresophagealesecton EEE ny abservation Explanation ‘arrats oesophagus oecurs due to gastre-cesophaal rfc ditease The intl sherapy for this conetionis PP-bated Usual, ‘once daly dosing of PIs preferred. ‘Observation onyis nota prefered strategy Baret's oesophagus is premalignant contin Hence. quckintenention is seeded. ‘The three remaining therapies are used in he NICE pathways in patents with moderate to severe grade dysplasia Aso, there isinadequate response to PPI therepy, then these modaltiesofteatment maybe Uied They are also used for recurrent disease Which of the following analgesics would be most suitable for the management of liver capsule pain? Pre ct an) Codeine © Dexamethasone Naproxen Oramorph Paracetamol Eee. © Codeine comes Naproxen ramaxph © rnin TY Explanation Contcosterids can be used inthe management flier capsule pain and dexamethasone i usualy the choice ofsteraid Pain sensitive structures inthe region of thelverincude the Iver capaue vessel anda tract. Stretching ofthe Iver ‘capsule bya primary hepatoma ce metastases within te ier can causechrenic cancer pain. Thiscommonly presents 3s dul, right-sided subcostal pain Referred pin atthe top ofthe isilteral shoulder occurs duet diaphragmatic eration ifthe superior aspect of the capsules volved Deramethassne 8-16 mg can be administered in shige or vided doses Ifthe sn improvement after fourta seven days of ‘use than it ehoulabe iscantinued by arcull suring the does ty 2-4 mg avery tara tofve aayewth the am of topping ‘completely. 18-yard manta ty tein phere presets othe Een Dprtretthtene thre ‘Poms carat nds onstrate ton ys cepts ‘oteineansete thar fright fun othe oprotmenn Hehontees tm ne {pac ofc rg pty test ce encore pe '9sonres:elotver So ebro popasesenon denn son lnesitions aw we 2210A, ro rnewn oe ese! ent iefonngitteret ean tare © cyumodnsinecton ° Eptanaervnsiecten © tain >| mesma | sot sn © | nner rein PanoniusB9 infection RE ubaliaintecton Explanation ‘The answer i Parvovius B19 infection. Pavovrus B19 isa single sttanded DNA virus andic associated with aplastic erin patients pre-canoee te haste snsems,nciting those witn ner spharoeyteee sey theta she nats reported here was B19 infection, which effects erythrocyte precursrshence its ably to result in severe ansemiain patients somes ed eal have shorter ifeepan than narmal Patients vith sever plate en ar likely to require cea transfusion ‘nti ther counts recover, which usualy ssocated with clearance ofthelnfection over a 10-14 day period. ‘CM. Epsten Bar and inuenca vius do notaffect te enthrocyte precursors, and are therefore not associated wih aplastic, ‘isis Although SU nfection presents with similar symptoms to pavowrusinfectin, isnot cassicaly associate with _2ngeria. nditisunkely in this patent gen he wil most ikely have Deen vacated Which of the Following drugsis ¢ P450 hepatic enzyme inducer? (Please select 1 option) Ciprofloxacin Erythromycin Ketoconazole Phenobarbitone Sodium valproate See See. Ue Erythromycin Ketoconazole Pnenobaroitone SEES Socium valproate Explanation The correct anewer is phencbarbizone The remainder of the listed options are all inhibitors of cytochrome P450. cranes © wersosee planation wasn srt pepo BRE 5 an ttn cused yaad epi ‘arty be nian dec ima, bate een oes Ren nena ee ot ‘Sasa sympomscan ange rm milsymgtonss cup nuies nd emg escches nus pans andes ‘sera srptonaauchssening Heian cues ey anaes enna Wale Ireaess me ingandcatae prea hemarngs nent neu eset purerayhaenhge seme fe tepateessongevonat hearers SS anda het tareset thai bday ‘vlan ress aspestepacundeetspatent eered wih eps undce SEER va peseras tre ster ester Nach DEI need Deiatheee8 sce aa, ers ake ate ovetnng condo ana eta agessey WV RtDEHES sypeMNetnepy ane asone stuatons os Eee aes tue prc donee cette ocean Which is the most common feature and clinical hallmark of carcinoid syndrome? Please select 1 option) © Bronchospasm } Cardiac valvular lesions © Cutaneous flushing © Diarthoea © Venoustelangiectasia COE Bronchospasm cade alesis i, TT Venous Explanation risodicfushings the inal hala GK occuring in 8% of petits mayest face och, 2 upper ches In severe casos shes are accompany aero in blood pressureandan increase inhear rate ‘The clea mantestations of RIBS VG ae dependent yon the combination flcsctve substances eared Che cf tha man fotures of rtrochromatin cose the production stage and secreonof serotonin When sectoin and other prodts are released nto hep ciculton they underahepaticrtabosm ac do ntusaly cause syste "ymptemscr sigs Hwee wth Ir tastass or hn prnarylsensin te bonchus and/or oats, te syste features of excinad sncromemantest occurs ines than 1240 stents tical wth cuteneous ushngoftheface.neckandupperchest Up 175 ofpatlntshaveartoes Lass en patente ny velop cara valvar bronmalies ily tng neat sie of te heart such as spleen cusp stnoss, Uma VragUGED and pulmonary tenet. Bronchoconstcion and venus teengectasiamay ao occur A 21-year-old anorexicis admitted for parenteral nutrition and has a tunnelled line inserted. A few days after parenteral feeding has begun you are asked to see her as she has begun complaining of diplopia, letharey and muscle weakness. She also has paraesthesia affecting her hands and feet. Examination confirms glabal muscle weakness and peripheral sensory loss. Which of the following electrolyte abnormalities is the most likely to have occurred? (Please select 1 option) © Hyperkalemia © Hypermagnesaemia © Hyperphosphataenia O Hypocalcaemia © Hypophosphataemia ae > types a) Hnocernins SITS Explanation “Ths stents sftn tom efeedrasndrome. vbar patents earls of range srmptemsvichcan cde + muri wesknes + pergeraneuropsty + neural erie and + myers dpraion Ins ehractenseay a ain phosphate, magnesium ptasium an ruc accumulation, ch neombntoncanleadto atest fomrefeedngincae those ah ae fedateraprolnged pre ofsanation sch as those mth aera smathote wh tress, ‘heey aoiangre een syarome na hypophosphatzenaisinvovementofaspacls etcan who wibesleto ‘seen atecuate phospnateelaceant dung nent saps oe eeang _ASs:year-old post menopausal women presents wih trednes and lethargy. she denies anyother symptoms. Merblood tests Teter: Hommagin ose, ors) Mean copescuariome ont @s0 Fert suet 0530 -Ant-ssue wansaltainsse Aan antibodies are negative. Gasboscopr and CT oknog phy re both permed an ‘do natevelonycaue fron dfiianey St icorimenceton ea ferous sulphate nd tvs mnths cher acriegabinis 1159/1 ‘ecm tose you tre mnths ater this iking wheter tie poset sop the raion se feseite making her -Aecodingto tei Society of Gastrontetlogy ues onthe managment orn deiencyanaemiawhatisthe ‘most apropate couse cracion? (Pease elect Lopten) | caste pear (© oneetansusin asreauired © reerortatnrinestaaion ©. step ocenanamantornaemogabin © Tralofateste ean preparation > cnangetapareneavon arr fortune evesigetan Stoponttonadmoniornemagbin erg © Talefstemstne oaltonprepsrten (IEEE Explanation Parenteriton placement shuld considered are tn tepacementneesaybutan cra prepenon cannot be tocntedorapierte There ienosndeatinfortrrcsionand hess been atespansetoiken supplmanttinthat ethelyta beep necessary. nshetunre, nat eestipatenstora gasrinasna case or tevon dtitenyhave een cagletag nde ngate the ahsence Sri tie Foe si Rec aap Ra invesgnion sno wararte.Furrermontringsreqied ensure hat ay response ssn. Suppementay tonino linger requdat his stagenthe patent's management so sicheg san akan not rewired, -Atematvepreparstanan dose eduction may/elp where alan not wet toate Se ‘Autorun nats Nn lechaistatohepatis ASH rom tot ES Peminysraingenoingte a) Explanation ‘ean pea any i (FBC Lehre ig te prominent ety sea PC ls mst common see me oped women eae are wth cnt oeinmine miedo The ezrton ‘tain reso coube wn semslerrsenALT ype hececten ans ceeeseteen to te a ‘umiasesne posbaryofeatesmmunogoDus a0 seen BC Araceae esenia section wthFEE tains nD riser Usoconchoiaiimay be ofvein vesting PCI ated vey exh bt time pets usualy eit er basa RSIS RAE es iy beute het esmaromated eon ane phomtoethamuenemalerree IMAL anenestate rams wousotns common NAS este ete gene eat na Mea Be cut ieenit temeen REATARD oC bet PSC morecrmneninmen doen contiaatonnaiatonmtay tows dseee Therese no ikactrt sage el epettreoated npn sty, anéamae prominent ein ALT woudbeespected ‘A58-year-old man presents to your clinic with dysphagia for solids for the past three months. He also complains of weight loss and loss of appetite. There is no other past medical history, apart from symptoms of indigestion and heartburn for the past five years, He regularly takes Gaviscon and Rennie tablets. He is a heavy smoker and a regular drinker. He undergoes endoscopy, which reveals-a small tumour at the lower end of the oesophagus. What is the most likely aetiological cause for the tumour? © Barrett's oesophagus © Helicobacter pylori © Oesophageal candidiasis © Oesophageal pouch Alcohol setscees | O Hoieobacter prion © Oecophagealconccass EERE © Besciphageal pouch Explanation “The history sugaestsa five-year history of gastro-oesophageal refx ‘Thiscan cause metaplasia ofthe oesophageal mucass, resulting in replacement ofthe rrmal squamous epithelium with ‘columnar epithelium (iso known as Barrett's oesophagus), which sa premalignant state Surveilance endoscopies are ‘recommended every to to five years, depending on the length ofthe Barrett's segment for metaplasia. The detection of ‘dysplasia prompts more intense surveilance ar therapeutic intervention. “The development of dyephagia for olde and weight loss suggests the presence of esophageal carcinoma. A 23-year-old man presents with steatorthaea and weight loss. On examination he is found to have a vesicular rash over his elbows and knees which he describes as extremely pruritic. Which of the following immunoglobulins is characteristically present at the dermo-epideimal junction? (Please select 1 option) © igs Oo oO O te eee ee. ¢ ‘ep ree ok ote Explanation Dermatitis herpetitormis is characterised by ipA atthe dermo- epidermal junction ‘Dermatitisherpetifermisis associated with SOBISGCISSAKE which isthe undertying lagnosisin this patent. ‘The rash which i pruritic and vesicularis found over the elbows, knees, buttocks, sacrum, trunk, face and neck Treatment is with dapsone and a gluten fie diet ‘Aseyeareldmanpresentswth aston of estonwich asben sea worsening over thepstew month He tes Yeurtathehasiostshainvourtn epasthaayen Teena pasted ofnte par fam meting 0cguet/dy Onemmintan hi Pie 151/80 ee else 7Sand veg SBMS Investig show oemogetin tose aes eee Barun suatow Mopnesumsteste Omeprazole anne © Upercinoccey Explanation ‘uideines rom he Department cf Heath suggest tht urgent endoscopy swaanted when theres indlgestion and weight los aay age combined wth araeria and voritng aridn te over 335 When there isaison/of tyr of pei and symproms ae contmcus Baum sow is ot the sul frst ie investision. wth upper pasties endoscopy prefered forthe mariy of paints, Magnesium isicateis on antaciduseorshort term eof symptoms ony anit predominant used now as an over te counter ingestion ret let ‘omeprazole standard phomcsloga therny fr BSDESAESUATSERSathovah not nan endoscopy hs ‘sxcuded a sencusunderyingdagnoasin ths cate 20-year-old man presents with anaemia, bleeding tendency, diarthoza, and abdominal pain. Examination reveals a palpable mass in the right lower quadrant and anal skin tags. What is the most likely underlying condition? Please s option © Chronic pancreatitis © Coeliac disease © Crohn'scisease Intestinal lymphoma © Ulcerative colitis, © Chronic pancreatitis cosine crohns see Intestinal mph | Ulcerative colts Explanation “crn disease commonly presents with diarrhoea, bdominal psn and weightloss Itcan affect the whole gastointestina ‘tact. the most common being leocoitis ‘Anaemia is usualy due t blood loss and less commonly B12/Aolate malabsorption. ‘An abdominal assis often palpable inthe presence af smal bowel disease which can lead ta Vitamin K malabsorpson. ‘Anal tags, issues, perianal stulae and abscesses are associated with Crohr'scsease and not ulcerative colitis ‘A S:yearaldweran comestote ini for review Shes been atering fom abcomina boating very strongly sling oneuriatn he BP 2125/82 mies enprtni37.°CHe BMl23/m anders sy Irestastnsstow t met cste0 on arvsta ox0400 ‘a s29rmeVt rune chore tlonnalsne mestbe agnosis? spent © wipttsdoeme 2 xcs REED Schistosomiasis © Shigetisis Tropical sprue © Whopescseee ERNE Explanation The history of abcominal bloating with intermittent claroea and strong smeling bowel gas is typical of slardasis. Aslongas ‘resh stool sample is examined, trophozcites ae Found in 602+ of samples [Asingle dose ofnidezole or course of metronidazole isthe treatment of choice ‘Adequate santationis the hey to reducing the risk infection although the infection rate rom uncooked foods is hgh in areas where Giarciais endemic. 55-year-old manis admitted with frank haematemesis. ‘The patients a poor historian butta recent discharge summary reports he was under the gastroenterology team two months previously with decompensated alcoholic liver disease. On examination he eppears anxious: he is tachycardic at 105 beats per minute with a blood pressure of 122/90 mmbig, There is evidence of palmar erythema and spider naev. Abdominal examination reveals hepatesplenomegaly and mildascites There isno evidence of melaena on rectal examination. He has @ further episode of haematemesis while in the Emergency Department which the nursing staff estimates at approximately 500 ml. Which class of hypavalaemic shack is applicable to this patient's clinical state? (Please select 1 option) Class! O | aassil © Classi Class IV © Cassv -\ oo xteston conten hontai hm ig Septem near ons ‘A 30-year-old male with a history of alcohol excess presents with a two-week history of confusion. Which of the following strongly suggests 2 diagnosis of Korsakoff's psychosis? Delusional jealous beliefs ° Epileptic seizures Impaired long term memory © Inventing recentevents Q Visual hallucinations Seurion ouster © prepueselues © Imparedton temmemary © netreccetoens ‘Explanation cS cm RTE, otemptoms mince * pty oe + Depression ‘aa + Pantera + Setimes bates rp but mare fequenty bisa eto with pan or ress perthelore newts ‘etexmenttinrvanue amin ane stententotne cneacunte of lone wan, Gaorealarinons voce EIT ae) planation Ansty afte area ayergamalewih aaemis mid asad RF andthe bow fndngroncalonsceybepH7 ‘seaaproscofuesatvecelt. “Teameat urn an ase ae tay supa lane ses flowed nrunaurpesion and gent ‘coms ceases cassay soc te wen Urs rem namin fsbe does ans oodestonNNEON costa asas doesnt present ody does Analog dagrossusay en psebe the duodenum é Bpsdrngcolnesopy ‘pps aseass cased by Tropheyma woe agra sostetatewm whch etsinmaasorpbon at nd ‘onret carinomas a on characte ppexance dso pete dug endoscopy nate oi would Decaagnasicarmaigarcy is amkuyraen Me aBesup Which of the following is mare likely to be seen in female patients with Crohn's disease who smoke? (Please st option) Earlier age of disease onset O Less aggressive disease course Older age of disease onset Reduced risk of requiring immunosuppression Reduced risk of requiring surgery Estee cnet TES Less pares dase course ‘toe age of cease onset © Recuednskoreaina mms son Sy Reaucea nek or aang surgery Explanation ‘ara spartan asseiaton benveen making and celina Conn dsease thas een shown te associated wth eae sae of onset of diese nd mre equent need for irnsnosugression ong women wih Crows dsease but not men, ‘Simokngs associate wit amore ages csease couse with patients mote hey reqieimmunosuppresion and surgery and snighrsk of ecurence ot seas folowing le-cxeal resection it isthoust that skin cou ater sooty muscle tone affecting endothelial function trough the reduction frit reduction or thatitafects the integrity of the gut mucoustarier. xis stress may so beacause as thas been fund that moncnucear clam smokerswith Crosdseasebutrot _ sents were ess senstive to ant-nfermatey protection against ave ieeraieal tress SER asocate th an increased sof Ueertne cots A 48-year-old woman complains of pruritus, steatorrhoea, and bruising, On examination, she is jaundiced and pigmented, with spider naevi and hepatasplenomegaly. Whichis the most likely underiying diagnosis? (Please select 1 option) © Alcoholic liver disease © Alpha-1 antitrypsin deficiency © Autoimmune hepatitis © Primary biliary cirrhosis © Wilson's disease Se | Ae! niin ing i a Pon ys isons ecse| | Explanation “Thiwoman has linia evidence of venice ets and portal ypartansion, “Theo main condone aug plementation nd conc verse ar pinay bia iE EC and paemoctromato Pacisa chronic cholestatic intarmatory Iver seat, the aetiology o which probably autoimmune. It mstcommoniy alec mecle-aged women “There isjauneice wth skin pigmentation sk of deveoning SERSSRSGEALVAERS and fat malabsorption ladngto decency ot the tamine 4 DE KGence ase andalso busing). ‘Sexumantiitochondl antibody s postive in 959996 cases Which of the following is a recognised cause of gingival hyperplasia? Please select 1 option) oO Allopurinal Hyoscine Penicillamine Phenytoin Prednisolone eee eee Py ee co enon ET ce SD Explanation ‘Arare adverse effect of alopurinal includes s metalic taste Hyoscine commonly causes a dry mouth. Peniciliamine can cause oral ulceration and loss of taste ‘Precnisolone commonly results in Candids infection. With respect to liver citrhosis, which of the following statements is correct? (Please select 1 option) © Endothelin causes dilatation of the sinusoids, thus decreasing portal hypertension © _Imend stage cirrhosis, liver transplantation is associated with 20% five- year survival © The final common pathway of hepatic fibrosis is mediated by the hepatic stellate cell Transforming growth factor is a potent promoter of the fibrogenic response by hepatocytes, © Tumour necrosis factoris an anti-inflammatary effector in fibratic liver injury eee eed _Endotpaln causes lataten of ne ssc. detetsing portal hyperanaon, Inenastage ross ver wansplatation assorted wih 20% ve yer sunal Thetnal commen pamnayothepatctorssiemedistedoy hetepstestee col ITT © _Tiansfomineeronth factors potent promoter ofthe orogenic response by hepatoctes ©. Tumeur necreisfetr isan ant-ntamimatery efector fort Wer nity Explanation Thehepatestolate cll esiein the space cf Diss andate cenzaltothe procs froin thelner Tumour ecrosistatoras apro-eannatay efecto bce enn, Prough actiaton ofthe stelae cel These cals ton secrete thelr colagenconstutingthe defn fates ofhepati fos, Intavuln-1Olethouphtt xerantnmmaten tees ontnetatte a _ngntnisavsotonsetor nthe hepa shusolds smiaryin he edotheum oF te syste ckeuato anaunctions byreausingconractin of the hepa telat cls husinereasnginrahenat srusislressance ndoromotn pal, hypertension, Nie ode atagonisestne tects of endorser. Fe-yea sural afterve transplantation’ new 73% ‘A 40-year-old male presents with @ six-hour history of profuse vomiting and over the last two hours had developed left sided chest pain and dyspnoea. On examination, he had ¢ pulse of 110 beats per minute regular and a blood pressure of 168/90 mmHg. On palpation, he had crepitus over the left supraclaviculer region and neck, reduced heart sounds an¢ left basal sided crackles, plus some dullness to percussion over the right base of the chest. Whatis the most appropriate initial investigation? e select 1 option) © CTwith oraleontrast © Echocardiogram © Gastrografin wallow © Rigid oesophagoscopy © Upper Glendoscopy a: epson TS oy TT Explanation ‘Tismannasahiton/tsvrevomitng niente poses caste. “Them renant nrg aneamnation th cep ce chest athgarpcalemphoema Teme pote ‘use esoremmcuriprect heonotan Moc ndbomiing espn andsugialemphysemicanies tant snot cae, ‘Trecresreymay corre salemanysens ‘icv sow corn te te of grerabonnaprdrstey 55-75 econmene st ne ‘neston Brum cme srt 0% ating oma peratensdutcarestenietasewre eury ACT wen cont con henbepeemadto conte atecpeerson swelasimipngthechersureaning Lara eck may buen te a7 Sager daeslsuneeran adage eons ot en ane esopageepyRassok ne Casogctn san Sree A 21-year-old man was admitted with confusion. He was noted to have Kayser-Fleischer rings and his liver function tests were consistent with acute hepatitis. Which chromosome contains the gene for this disease? Chromosome 13 © Chromosome 15, © | Chromosome 17 © Chromosome 22 © ceenasame rs Expanation Incipio ceeoentocansgosmn (eos oie ce ct AE a oo competed os Pama threes den enh aor cnr Inet pater plana cau i (+20 m/e coppers moan Spent ‘sent ne penning tc ctoenay pert nee ena ces MBE re cacy he ubtston fresno apse 22 he ta snc cotinine energie edn censure aotbomstas ype sete amutson er setenotne N11 lees onchonatone Naoftomstais pe 2ithereitcForutorordetionltheN2atnetendonchemstone 22 Which of the following is not true of a patient with ascites due to liver cirrhosis? 1 option) © Cardiac output is often elevated © Hepatic intrasinusoidal pressure is elevated © Spontaneous bacterial peritonitis is a recognised feature © The usual source of the ascitic fluid is mainly from the exudation from the surface of the liver © Urinary sodium concentration is usually less than 10 mmol/I © Cardiac outputis often elevated | Hepstcnasrandspesnescot © Spontaneous bacterial pentonitisisa recognised feature: © Urinary sodium concentration isusually essthan 10mmnol/ (gaan | “Te usual soure ofthe actif main rom teexucstion om te surface of te iver Explanation Hepatocellular failures associated with hyperdynamic circulation and systemic vasodilatation with increased vasculer ‘capacitance, Most pationts have sodium and water retention, © ramen EB pation ‘Pees sutratncien cg ep ce Pease ajanon seit gen cece ure Orisa exoxsynostny eters sitanretnetotinetngeah neti nigirreapucnaesn uate gieincn amet ny ana a (pegeeneovapsonay ests enpepangiquconeiore neato Rasta ‘rem upinowmhne emanate nese ste prunctgtocecpaeca asa carmen droeso nie eayin md cieobteasenenmanibemcnids A 24-year-old woman has ingested an unknown quantity of paracetamol tablets four hours ago. ‘She now presents with nausea, vomiting, anorexia, and right subchondral pain. Which of the foliowing features suggest that she should be transferred to the liver unit? © ALT800 units/L © Blood giucose smmol/L © Heartrate 120 BPM ©) pH725 © Systolic BP 100 mmHg becca ail ‘ALT 800 units. Blood gucose'S mmol ee a) rs © Syste BP 100 rae Explanation ‘ApH oflessthan 7: is poor prognostic factor fortis patient. ‘Te cela for transter to aspecialst untae: + encephalopathy + INR:>20at «48 nous or93.5at <72hours ‘+ serum creatinine: 200 prov. + blood pt<73 1 systole BP <@0 mnie at-old woman is admitted with headache and vomiting, She denies abdominal pain. She reports weight loss of one stone over the last six weeks. On further questioning it becomes apparent that she has noticed the vomitus contains food from several days ago. Abdominal x ay reveals a prominent gastric bubble. Which of the following is the most likely diagnosis? Pease select ? Acute cholecystitis © Colon carcinoma O Gastric outflow obstruction 2 Peptic ulceration ° Reised intracranial pressure b—--—__—_—_— —- peutecoleesiis Colon carcnem © cexticoutowobstucton ES Peptic eerstion oised intracranial pressure Explanation Vomiting of food fram severe mesisago suggests gasses or gsc outhow obstruction, Inthis ete wth the stony oF weight ss, anundering malignancy suchas artral gate carenoraisikely Patents with SSUES rypicay nave ab domnal pai. usualy nthe ig upper cuacrant or epigastnum. Nausea ‘vomting may also occur Patients are ofenfebrie ‘Aodorinal pan and changin bowel ns ae the common nical presentations of colorectal cancer poe abdominalpin or dsc te mest prominent symptom pate wit ast eae occasional there maybe voring, Circa Features fase intracranial pressure include headache, reduced consciousness and vomiting Sgnsincde sith cranial nerve palsy and papiloedema Hypertension and Bryer (Cushing's reflex may sso bes2en, A 34-year-old man with ulcerative colitis is admitted with severe bloody diarrhoea. He is opening his bowels approxmately 15 times a day and has abdominal pain. His current medication includes Mezavant and on admission he is commenced on intravenous hydrocortisone. You are asked torequest a thiapurine methyltransferase (TPMT) level as the plan isto start azathioprine ata later date. What percentage of the population has normal or high TPMT activity? ) lect 1 op © | 4% 10% O 2596 Oo 50% O 90% — Explanation ney percent oft populton hve norm righ enn setts tits arehenenasurfrtewietypealie "ci se enya TPMT re tant ep ern. ‘Ten pe ceo papuatennavenect eves ofTPMT acy, tai on lo ype anc one var ate. nein 900 people fnctenalenzme acti ‘Several goupsofpasents- not ortythase wth inflammatory bowel dsesse-have developed aathionring induced myelosuppression inked to TPMT deficiency, With respect to gastric carcinoma, which of the following statements is true? ‘ct 1 option) O Aspirinuseis a risk factor for gastric carcinoma Early diagnosis of gastric carcinoma results ina five-year survival rate of 20% O Endoscopic ultrasonography is superior to conventional CT scanning for local tumour staging, Helicobacter pyloriinfection is not associated with gastric carcinoma Incidence of distal stomach tumoursis increasing spin sais a sktactor for gasticcarcinoma Ey anos of psc carom ecuts in fe your sil tof 20% endzone utasaner apy spear te convention CT zcarring fr eal tour stg © Hetcotscerprorinfectonisnctasocaesuihgstccacnons Incidence of ital torch tours inten Explanation “Thsincdence of tal stomach tumours isactaly decreas while theincdence of taursin the roma stomach sreseng [sau isassocite wih decreases of cetin gabe tumours porinteton hasbeen assoied ina nurber of sues wth increased iskef gate earnoma aneaty tage andinshited ands, ve Scening fr gastric catcnomain open detects upto 4% of gsc anos year sunalcan be upwards of 50%, coth gsc tan sa use complemortary vestigation inthe staph of gasticcarcnoma but escape ‘trasonogipnyesupena to convertional CT eanning aitisableto assess cet cinaiaten and ympnatiesskmination oftumaur ‘69-year-old man is seen in outpatients He reports weight oss of 1 stone over three months but his history is atherwise unremarkable (On examination, his abdomen i sft with no palpable masses. APR examination is normal His blood tests show Haemoglobin ses 120-160) ey On (60-96) \Which ofthe followings the most appropriate investigation for this patient? select 1 option] © Abdominal xray and colonoscopy CT sean ofthe bomen andé upper Gl endoscopy © Sigmoidescopy and upper Gl endoscopy © Ultrasound scan of abdomen and colonescopy © Upper Glendoscopy and colonoscopy Abdominal xray and colonoscopy © CT scan of the abdomen and upper! endoscopy (©) Sigmoidoscopy and upper Glendoscopy > Ultrasound scan of abdomen and colonescony. © Upper Glendoscopy and colonoscopy ED Explanation This man has weight loss and an unexplained microcytic anaeinia, ‘The likely site of blood loss is from the Gl tract in absence of an alternative explanation. ‘This maybe due ton occult Gl malignancy and therefore the initial investigations of choice are upper and lower Gl endoscopy. Drug induces colitis (© Faria adenemataus polyposis FAP) Hereditary nonpolyposis colorectal cancer(HINPCC) ee) © Meocpe cons Explanation Melanosiscolis benign eanition associated with chronic lative use, most commonly osmotic lacatives. The pigmentation {sdetolpotuscn formation and accumuitionin macroohages ts most commnonyisentied at endosoopy and eauires ‘reatmentother than counseling on laxative use ‘The LiL and hypokalaemia are likely a result of chronic darthoea, which hs ls contibuted tothe weiahtloss. ‘Microscopie and drug induced colts would not present in such a way with bloody painful stools tobe expected Both FAP and HNECC would present with numerous tumours within the bowel lamen and usually family history of wel ASB jrldmanon na cut teament aris quescent eet cots) sfundtohaeanESRof9S mm/h 1020 rav'istoun nests stor Heemogatin way wee ase ca asa tes vost ra tet cians. soremarowtehine ndaseraion eee hay ieeat sine 930 Bone marroivtrephine and aspiration \sotapebonescan Fomine: ED Rectal biopsy © rRoyskeeta uvey ET Explanation “The erythrocyte sedimentation rae ESRD snot raised in quiescent UC. Hence there must be anther reason in this case, ‘The only abnormal result gvenisa raised IgG. This suggests that myelomais the diagnosis Plasma immuncelectrophaceséto look for an M band ithe most appropriate nest investigation. | bone marrow trephinets the definitive investigation but itroumatic and painful to the patient and zoienot the net investigation of choice. Which of the following demonstrates autosomal dominant inheritance? (Please select 1 option) O Acute intermittent porphyria © Cystic fibrosis © Dubin-Johnson syndrome © Haemochromatosis © Wilson's disease ‘nation aun snssnn nt Cnatenesensineaan scat ietwiom mctnre ements rh a ane ta warn eu RON ae ‘SERENE ace nis ersten et ie inp ‘Meese ere atnosieceanenerietnererominnietcrn meee ‘30-year-old man presents with acute, profuse, watery diarthoea with some blood after retuming from a holiday in Tanzania. He had been taking oral rehydration salts. Which one of the following is the most appropriate treatment? jon) © Ciprofloxacin © Loperamide © Metronidazole © Prednisolone © Vancomycin Loperamide = © Pehscene TD 8) momen Explanation ‘The most likely cause of such traveler's diarrhoea is Escherichia colland hence ciprofloxacin is fecommended for fst-ne antibiotic therapy (when naeded before stool culture results are available. Metronidazole would be suitable for Giardia infection but its course is usually more insidious. Which of the following genotypes is associated with the lowest levels of alpha-t-antitrypsin (AAD? (Please select 1 option) © PiMM © PiMs O PiMz © Pisz PiZz } See ee | isan Explanation PIMMis the normal phenotype. The null phenotype (not inthe lst of options) isthe least comman but the most severe form of the disease where there is no detectable AATin the serum, 34-year-old woman is referred to the gastroenterology clinic. The GP referral letter states the patient has persistent lethargy and blood results have demonstrated iron deficiency anaemia Which of the following statements regarding iron deficiency anaemiais correct?’ select © _ Endomysial antibody serology to investigate for coeliac disease should always be requestedin addition to duodenal biopsies © Low serum iron with a low totaliron-binding capacity confirms iron deficiency © Microcytosis may be absent where there is combined iron and folate deficiency ° The presence of HowellJolly bodies on blood film would go against coeliac disease © Thrombocytosis indicates chronic blood loss Ears onto serology to imesigat fr cote seas shoul aaysberequesedin atone duodenal Dopsies Low sunken wth alow talon ing capt coms on tency © meiopossmaybesbintwneetneeieconbideon anatase ensy The presence ons oode onload mak go nga cone ase Tamaeytessinacates can od oss Explanation luo defieney type ests na mircyoss but this canbe absent when the is concurent SIRES tien ‘ypcaly results ina maroc RAR se cause ton deen in 2K of cases, and neodsto be nclude. The gol standard for diagnos are ital vaderal 02 biopsies. Endomys antbotyselegy cane weft confirm te uote lous aeohy ise due to ‘moter cause faa sceenng esprit encoscopy. nobody tres an ao nap to determine wheter oc neta pale is ‘stheringtoaglutan re dit However itis not tut ay ht sarlogy shoud ALWAYS be requested wth opis Low seruman Ceo pnal/L wt high ttl rnin capacity 670 pmal/U cents tn dingy, Howetolyboiescn lod i nccate hypesplensmand ae consistent witha dasrosscfcolac dese, “Thvombeeytocis may occutin acute raemerrhage bits also eundine numberof oer stuatens uch asinfametory condone. Which one of the following require urgent referral for upper endoscopy? lect 1 option) ‘A35-year-old male who has a history of waterbrash and dyspepsia which has responded to 2 course of ranitidine but since stopping has recurred A45-year-old male with a one month history of persistent dyspepsia © A56-year-old male with a one month history of dyspepsia and a pulsatile central abdominal mass © A 62-year-old male with a three month history of unexplained weight loss, tenesmus and a right abdominal mass © A73-year-old mele with a three month history of dyspepsia which has failed to respond to a course of protonpump inhibitors ‘Asyearclamaewnonas ais orate ar pepo wens respndsto acouseoanne at shee swopnanaseores -Aesyaurtamenthsene rem ttn pean pena ‘seer lomae wna neem nine aspes8 a pute cena oma mss © Aezyeorlamae na tee ran se ctuepane wets tne nd 9 Hebert ‘Artyeuclamienthatiee mons pepsi ha ad esantoacou of pion pum tspanation + Dppapattenyaes) * pepsi atany ae combed wih anyone of ets, anaemia oom + Dppesainapwenatd 5 above mont pepsin ony ad ersten cnpoms * Djpepiawineneottaretseesphogs any hf ep aontestalG arnoma peices aaem s fancee Win rgncto the presmredanseyearemanhaegpepa wate te nari ane T Inthe cae newpinedwepht os nesmutanduppe gt mss theprcblemiskely tobe acon cwcho Which of the following statements is true of autoimmune hepatitis? Pie: ° oO ° ° ase splect 1 option) Itis associated with hypogammaglobulinaemnia it may be associated with keratoconjunctivitis sicca It rarely interferes with menstruation except in later stages Itraraly presents before 20 years of age Itusually presents as an acute hepatitis | e | IRisatsocsted th hypogammaglobulinzemia emaybessocstedwithkeratoconunctnits cco EE tesarelyinterferee with menstruation exceptin later stages © teyprsenscetre20yeusc'oe Emme = Explanation ‘occurs fequenty in young (10-20 years) and middle-aged women ‘Twentyfve percent present as acute hepatitis but usually the ansat i insidious. Some may be asymptamatic fr yearsand them are found to have signs of chroniciver disease. ‘Amerorshoea is common It's associated with hyperglobulinaeria and other autoimmune disease ‘Shy percent ate associated with HLA-GB, DRS and Da The sicca syndrome (erostomia/dry eyes, erstoconjunctivits scos) may occur. A 65-year-old man was investigated for weight loss and dyspepsia. Endoscopic examination revealed an ulcerated lesion in the stomach and biopsy revealed the presence of a low-grade mucosa- associated lymphoma (MALT) with Helicobacter pylori Further investigation with CT of chest and abdomen were normal as were bone marrow aspirate and trephine. Whatis the best treatment option for this patient? Please'select 1 option) Eradication therapy for Helicobacter pylori ON chemetherapy Oral chiorambucit O Partial gastric resection Radiotherapy Eradication therapy tor Heicobscterpytor Aer Wehemotherapy © Cnlchcnesct SERIES Pita gasinicresection == Explanation “This isa gastric MALT tumour ‘These are usualy marginal zone Bel ymphomasand associated with an excellent prognosis Low-grade gastric MALT tumours assacited with Heficbscterpyorinfaction respond in over BO% to Helicobactereradieation asthe primaty mode ef veatment, ‘Radiotherapy is considered butis genesaly unnecessary For the diagnosis of Giarcta, what is the specificity of ELISA against Giaraia antigen 657 (Please select T option) 280% CO 388% O 290% OO 395% 29856 st om . 9% ED alinadon ‘Several mmancasays using abodes against esto rophozoteatigenshave been developed frstol naysiin te \daprossc Gare Alstieisinchde rect mmancfuorescent ssys(OFA munochromatograhic ss, and eneymetinked immunosorbent aay (ELSR. In gener ese methods have rete sentiey and faster tumaround tine than commentional stool mrscepy metas. Speeety and ost are usvayreatvely compara. na suey of tol sale fom patients wn abdominal symptom king erent sys for detection of Ciara, the pesicy fa number oferentimmunoassna as 268% ELISA uses he nenuncogial pncl of n antigen binding tats speshe anti tety alntng ne detect ata quan fanigenssuch sores peptizes,hamanes. rate inafuidsample The asay ties eran abla antgons and antbodae to dteerbologcalmoleces The antigen i alowedtobinto aspect artbody. which subsequently detect a acondaryantbody tat scope to an eye Avis colour change or fuorescence then indcatesthe presence of ancgen which allows quantativeor austatie messes. ‘A 42-year-old man presents with a si-month history of diarrhoea and abdominal cramps. On further questioning. it appears he has also noticed facial flushing, On examination, there is wheeze on auscultation of his chest and abdominal examination reveals hepatomegaly. Which of the following investigations would confirm the likely diagnosis? ect 1 © 24hoururinary 5-HIAA ) 24-hour urinary copper © 2é-hoururinary free cortisol ° 24-hour urinary protein ° 24-hour urinary VMA uration ‘hsptenrmccretanone Te are may 2 utrecsncin ke hrtoundoeci SH funy a pncetfescragtiaurtnte pdt Naeem nN ecm ote mE mets Se ann papi anc abe cat etre ESSE int di coro n t ‘resp pomonay rere ‘etl oonon pean! tepan rear szombememecie conters tater eee penta ne ery arate 3p nose geen. 2¢narvenayreecomatnune omen tts ipernity ace hae pores ing rE nb ptemady 2 esa 4.0 tym A What proportion of patients diagnosed with primary biliary cirrhosis (PBC) are positive for anti-mitochoncrial antibodies (AMAI? Pleas 1 option) © | 25% O 45% O 65% O 83% OS 95% anaion comand rte senegal in ESA ee (iit ee nei 1 one arp natn yp deme tesa Which of the following stimulates the secretion of gastrin? (Please select 1 option Amino acids © Fasting High level gastric acidin the stomach © Low gastric pH Somatostatin © xno ET sone > High evel gastric acid in the stomach © tovestic os SM Somatostatin Explanation ‘Gasttinisreleasedtfiom specialised endocrine cells, called G cel, n esponse toa mel, Protein, peptides, and amino acids are specie components which wl stimulate gastrin release, Fasting and increased gastric acid in the stomach bath inhibit the release of gastrin. High gastric pH is strong stimulus forthe ‘secretion of gastrin. {Goals are tighty regulated by two hormones: gastin-teleasng peptides a stimulatory effect causing the secretion of _2sstin, while somatostatin is intibtory. A 35-year-old woman was referred to the gastroenterology clinic with rectal bleeding. She underwent a colanoscopy which showed dark brown pigmentation of the entire colon, along with a small polyp in descending colon. Als foun. intemal piles were Further questioning revealed that the patient had a history of chronic constipation for which she was taking some form of herbal medicine for long time. Which of the following histopathological picture may be found in colonic biopsy of this patient? (Please select 1 option) © Fungal hyphee in colon Increased melanocytes in colonic epithelium © Macrophage infitration of colonic mucosa lamina propria with lipofuscin pigment inside the macrophages ) Malignant changes of colonic epithelium Malignant melanoma changes of calonic epithelium Frgatnnion can Inressea metnoeyesincoonc ep acesogentzaton fon: mucosa imine ora weh pein lameince he MaoRhaES © Nolpencrnesscrecnconehm Samy Mulgore metnons rungs colniceptelum Explanation “spt it ity of nate ste shows ie pigmentation of len This sggene of moss heey consequence eee abo ataqunote aes Chen we of oquene mates case ye ‘ne caeniceotetum wn goneratn ct pesch pigment Ts parent subsea ergy te macopasesto Benet henstogcal pure nw, tee cangs re eesbe ane opp ease may ead |darpeennceatthepbrctipechunges ‘Tre mace nates such sigur na payee shld netcause hee crangs Howe beste ‘ped! ge cota ingredient ne cacattencontananbaqunene ‘obtain doeseaserns peo cobrexcen aneaanc, Inmlanois cal tect need melancyts bua esd bnomalpeusn gent ign changeset conte nu sen asia roth erphpial mas Dee igmesin inti Men cobn avery Ul, pemant alae many vcr ary tbe gente Thea Deaaintsseenthasne elton tomdanos cok Menoss alan anton condone dagrsed ‘nedelyonclnoscony.Thebleearainanscae casey the eps oF OY. A patient presents with haematemesis, An oesophagogastroduodenoscopy detects a bleed in the lesser curvature of the stomach, Which of the following arteries is most likely to be the cause of the bleeding? Please select 1 option) © Leftgastro-omental artery Pancreaticoduodenal artery © Right gastric artery Right hepatic artery © Splenicartery Left gusto omental atey Pancreaticoduadenal artery ence | ee Splenic arery Explanation. ‘The pancreaticoduodenal artery supplies mainly the upper andiower duadenam and the head ofthe pancreas. ‘The gasto-omentalatenes supply the preater curvature ofthe stomach “The right gastric artery arises from the hepatic artery or the left hepatic artery supplis the pylorus and travels along he lesser “curvature ofthe stomach, supplyingit, and anastomating with the let gastric artery, A 49-year-cld female presents with a six-month history of pruritus. Examintation reveals jaundice, xanthelasma, scratch marks, vitiligo and 3. cm hepatomegaly. She was afebrile Liver function tests reveal raised bilirubin, alkaline phosphatase, gamma glutamyl transterase and mildly elevated alanine transaminase and aspartate transaminase Which of the following conditions is most likely to be found in this woman? (Please sel © Constipation Haemolysis © Lymphadenopathy Vitamin A deficiency ) Vitamin B complex deficiency a Hoernolyss Vitin Aeconcy IB = © | monn a | hema Explanation. ‘The most key dlagnosisis primary biliary SRNR as evidenced by rusts + jouncice 1 raed ALP, and A 61-year-old man has a 2 cm adenoma removed from his sigmoid colon. The biopsy results confirm an adenocarcinoma in situ with moderately differentiated dysplastic cells. The pathology report Confirms total excision with clear resection margins. What is the most appropriate follow-up management for this patient? © Annual carcincembryonic antigen (CEA) Chemotherapy © No followup ©. Regular follow up with colonoscopy © Regular follow up with no colonoscopy o Annual carcinoembryonic antigen (CEA) = = Regula folow upwith colonoscopy EE Regular follow up with no colonoscopy Explanation “The rs thing to notes that the queston i about planned management “This patient hasbeen picked up early and has had etumourresected HisCEA wouldbe normal and woud not be expected to be elevated until the cisease wos quite estalshed onthe TWM scale. However. ths patients prognosis would be excellent but heis by definition someone with increased risk Therefore he should continue to be reviewed with colonoscopy annualy for atleast two years ‘tena peo0 a samen soe anne tanes Hnagi cht flown est ny ars ° ° ° ° ° ‘sen oeerven rane roy rose opmaut omer ‘Active Crohns disease acteral overgrowth syne deat IT Ischaemic kts © Shortonelontome CEM Explanation ‘Wie are tol that this patient has undergone extensive distal small bowel resection but she doesnot have.an ileostomy. AS such its most ikely that she as colonic ble acidiriaton leading to diarrhoea SAP BSWaLSYRAHERREc usualy associated with voluminous darhoea ef upto sires per day, normally through te ileostomy outlet “Theteisno indication ofective Crohns seas, and bsctesa overgrowth syndrome is usualy associated with symptoms of abdominal blasting Cholestyramine may be effective forthe treatment ofbile acid darhoes, A 28-year-old woman is referred by her GP to the gastroenterology clinic after a recent gastroscopy with duodenal biopsies confirmed the diagnosis of coeliac disease. Whatis the prevalence of coeliac disease in Europe? 1 option) OQ) 410 O 430 O +300 © | 11000 +3000 Enpianation| “Tre prises REE Erne vars woRy ncn te epon between T1031 1300 ismerecommon samecenepepusen ea ice aT cel metineclypesanstyeatinte hen chase sin infarmstion nd ‘sop presenta ay agen ane cormorant pe presen 2s 0s Wornateslahy mote ‘omeoni ate. Fates syne oes anastextores aye pet. Thetlseten3 mls macy anemia win ote ‘casey enor eo sn, Te os pais ia A Pg caleumtarin a nea aoseen, eran ene ancant-suetrnigramnaeatbedes ose sop est tendompsanodes re ‘0x sense andes 100% spect opis Mon ne secn parte be underum eset an cemonste bus sey. rage inciecautenes det paens shuld ain ey nde A52-year-old male is admitted with vorniting and acute epigastric abdominal pain which radiates through to his back. Investigations confirm severe acute pancreatitis Which of the following figures most accurately reflect the mortality associated with severe acute pancreatitis? (Please select 1 option) © Lessthan5% Approximately 109% © Approximately 20% Approximately 30% oO Approximately 40% Less than 5% | hse | cae] ‘Approximately 30% S| memae Explanation Mortality in acute pancreas varies according to age, comorbitities and severity and is scored through the Ranson scoring system, However, average mortality with severe disease has remained pretty much unchanged over the last twa decades, andis ‘approximately 20%. ‘A 65-year-old man is referred with abnormal liver function and undergoes a liver biopsy. Which of the following count against hepatic cirrhosis? Please select 1 option Fibrous septa formation QO Granuloma formation Liver call necrosis ° Nodular regeneration ° Subendothelial Abrosis pacer. i (Fibrous septa formation contonstonsten ESS Liver cell necrosis ° © Nodutar regeneration © Sbenichetl ess Explanation Granuloma formation i not classically seen in fitHBS, which can be micro-or macrenadularin type. Inthe micronodular form, the nodules are less than 3 mim across with uniform iver involvement seen in alcohol or blary disease. Inthe macronodular form, there are larger nodules, classically seen in chronic viral hepatitis, A 17-year-old man presents to the Emergency Department complaining of intense pain an defecation, which persists for some hours after the event. The pain recurs with each bowel movement, and is so severe that he is now scared to defecate He is passing harder stools over the past few months and has noticed fresh blood on the paper and occasionally even drips of blood into the toilet. From the history, which of the followingis the most likely diagnosis? c select 1 option) © Anal fissure © | Crohn'sdisease © Irritable bowel syndrome Rectal carcinoma © Ulcerative colitis ote CE Crohn's cisease oo ncn Se Rectal carcinoma | Ulcerative cottis Explanation The historys ofan ANAL SSUEE, which arses as a mucosal tear folowing passage ofa hard stool Most ana fssures cecurin the posteriormidiine, ‘The majority of cases respond to conservative therapy which ineludes stool softeners and bulking agents. Topical GTN treatment may also be useful for sphincter relaxation, Surgery is reserved for resistant cases. A55-year-old male presents with dysphagia, retrosternal discomfort, and weight loss. Studies reveal achalasia. Which of the following is most likely to provide symptomatic relief? (Pre: jon) selectt © Buscopan ° Diazepam © Nifedipine Omeprazole © Surgical cardiomyotomy reso ee) {planation tex cam cna Deeside andthe ower epogelphncec Hove De eMEtoN ‘sympensirbe steve anc si soap Use ted aves es Creat. meet ‘tapped attra al aris ne a stant er iar re “tts org tne fie media hepa pic ama nunca andtedo rama ot “nintnenton Sc etme omar paver wth dente bteen 5). Nene Rowe ‘wade tllampamirgvenent ‘onemome ont eceres nome sudes tua ocd Per mony show ice exter emBton ‘epee ino patents Cer studs tual tednaues Sow Seton espana 84-00% senetseenso psec atin Yue bjondes aoe erase ecrenceRenusthe mst ‘ommend encanta oceare Mens ntoeston maybe erates soapstone we asphalt arent wth oder Hore reece shried (ype tancamon ose peel ecermeneio rested to tefl aly natn arsine veep pseshan ok Nene fete cokes hep hesymptans cesophages dataton is usvly atterpmd before aptocopc sural moto. ‘A.21-year-old woman presents with flatulence, bloating, loose stools, and diarrhoea for three months. She takes no medication and has no allergies. Avoidance of wheat, dairy, and gluten has not improved her symptoms. Colonoscopy reveals nothing of concern, Which is the most likely diagnosis? Pease sek Coeliac disease Crohn's disease Irritable bowel syndrome (IBS) ) Laxative abuse } ° Ulcerative colitis ee rotenone ss —- Explanation las scnoncrapseremiingconation ncn seal cto ang chargeinboelhatit ndstcontlpitich snorresminceteaion oedema aes women" FM The elegy sete Undeoed BE seeMstS| "ove onrma neo muscle act ance persesenysnddneral caw ocesshgot MSM. Is _soasncneo wn cenedloesaenate des andpoteeorasteB es. Dares sbsedonencson other bowelpsnologes ant ctncaleaninste aniston Texmentbisedon day ‘range pacoegl super nd eceaten ot Eee. ‘cons ate nazommune edie sce nics Tel meated cag does, mts nd pe ta ‘es anc sus ean at yen foe fom mouth as eae cats roi presents wth eo pil arboes “hereerahetry sgt nate stu nd clectap wou ty snow nec ona nse ou hel morveyremeving en and whet mth die ani ignsely anceps ‘A 64-year-old lady presents with symptoms suggestive of irritable bowel syndrome. Which of the following would represent a red flag! incicator and prompt further investigaton? Ck Arecent change to more frequent stools in the last two months © Abdominal pain Bloating © Change in bowel habit present for at least the last two years © Weight gain Arcentcarertomoereuetscobnieletio ments Se ating > Werte Explanation “Thera ndicatos ested nthe NOE ais + wenn andunmtenceawegntos + recatesang + ation oSowe or SESS a + serangein soe! to loerona/ar mere agente pertingformoreshan stwestsinapeson aged ‘so nclricaenaminaton tethered finds are + spranalmass + intammatary maser for ntammatery done eee A37-year-old woman gives an eight-month history of bloody diarrhoea. On average she has six bowel motions per day with associated urgency. She also reports weight loss of approximately 3 kg. On examination, she is dehydrated and tachycardic. There is evidence of pallor and abcominal examination reveals left-sided tenderness. Which of the following pathological features would you expect to find given the likely underlying diagnosis? e + 1 option) Diffuse mucosal inflammation © Lymphoid aggregates Q Normal cryptarchitecture © Presence of goblet cells Q Transmural inflammation emesis TD tympnoidaaregaas Noma cop areitctire © Pocencectecbetcets Sem “Tans intammation Explanation ‘Crone garicuay for this uration bloody danhieainayouns patente very suRsesve ofa agnosis of ueeratve ott, ‘Thetime cours ito long france eauee and ntl roms may cause colts, tila requent. Booty darhoes ocurs30-100%oF cases of UC Cis seas ten presen wth ineitient abdominal psn dares is usualy watery andin hal ofalleases presentation swith erianalsease. Bloody arinoea may bea feat of Crohn's ‘oistutthsisless commen than UC. The predominant sed symptoms wzency and anatomical distribution ofthe ‘tendency sould ato point towards UC. ‘The answer hee, erfore is cise mucosa inrammation al theremaning options are stolen estres fiche esease Mucosal ntanmatan's one of eistolegel Nang IMUC, uansmuralintanimaton Seen Conn aess8 Lymphocytes ae seenin robs disease, whereas neutrophisarefoundin UC ‘cpt anecestes and goblet cel cepaton ar so found nC. Which of the following drugs dees not undergo extensive hepatic first-pass metabolism? Please select 1 option) O Budesonide Carvedilol Ketoconazole Salbutamol ° Warfarin ° © udesenige D | Carvedilol Ketoconazole salbutamol © Worn ES Explanation The correct answers warfarin. ‘The remaining listed rugs all undergo extensive hepatic first-pass metabolism, A 43-year-old male presents with weight loss and watery diarthoea. Investigations reveal hypokalaemia with a pancreatic mass. Which of the following would support the diagnosis of a ViPoma? (Please select 1 option) O Achlorhydiia © Hypogycaemia ‘© _ Increased pancreatic polypeptide Migratory erythema O Pellagra hone I © | Hypoatyeremia (© Ineressed pancreaticpalypeptide © Migstoyentnems CEA) Polagra Explanation Lit casscayassosntes wih DI together wth prtuseiarhoes a hypokaleric acidosis and ypergycaemia. Mirtoryerytnema associated who lucsgonoma, Atnough sed pancreatic plypeptei seen wt MBSE itis unusua an ismore comment associated with sown syedrame Palopais ssocate th the arco syndrome _A32.year-okd woman has presented othe Emergenky Department forthe tied me wih abdominal pan which pKa sts ‘eve ys. Th reous wooo seas be pert soymnomte econo feign Seuss estatea we cons nate pate corpo dense nome tet damoss Been ences ‘neumnstonseaneousana pers a 15998 me wihapucoT 102 Seas ashes He seonen aac mage rasa, ecatcount anata Pts aon nenornerawrgsemstbe Samos? © etemtent roots © mi © orion © eriscraes tas © veg pig Acute niemitent pornyn Ne pepe Depression | Porpriyiacutanes tarda Voiegtecortyin Cer | Explanation Ae AIP is a rare sorder characterised by abdominal pain and neuropsychati symptoms which usually presentsin the 20-40 age group. Rash isnota feature of AIP, unlike other porphyrias. Hypertension and tachycardia are commen examination findings ‘The progesterone component ofthe combined aal contraceptivepills known to precipitate porphyria attacks Many other drugs have been described in association with an attack, hot Aono disease ia OS Nenakonoiestatchepstcs Ce Primary biliary ermosis Vorotts My Explanation ‘Thisis acase of non-alcoholic steatohepatitis, the diagnosis of which is made only by histology of ver biopsy which shows lesions suggestive of ethanol intake ina patient known to consume less then 40 g of alechol per week, ‘The dsgnosisis supported by the presence of obesity hyperelcaemia, nd hyperechogenic hepatic parenchyma. Inalcohetc hepatis. the AST is nomalyased more than the ALT typically wth ratio of at last 21 A 24-year-old woman who has a long history of ulcerative colitis and takes meselazine 3 g and azathioprine 125 mg per day discovers that she is 10 weeks pregnant Shes also a smoker of 15 cigarettes daily. She attends clinic seeking advice on the menagement of her medicines during her pregnancy. Which of the following statements is correct? (Please select 1 option) © __Reathioprine can be used in pregnancy without significant risk to the fetus Initiating an elemental diet predisposes to fetal malnutrition © Mesalazine therapy should be withdrawn © Steroid therapyis contraindicated © Termination of the pregnancy is advised © Auatioprne cnteusedin pregnancy without sgnifcantrsktothe fens EER Iniating on element cet pressposesto ea manuttion Messlapine erapy should he wincwm © secs sommnecoee ‘Teamination ofthe repnancy isacvised Explanation -zatvoprine use forthe testnent of ifanstony bowel disease in presnancy snot esscieted wt sine increase ‘compeations of pregnancy, nor fetal mafomatios, ants usin his setungis considered sate, Animal tues suggest an increased risk flow ith weight but tis isnot supported by the dt avaiable humans. The ik ‘othefetus fom uncontroledinfammatory bowel dseasels significant and mairtenance of remission is ery important ‘el cones EERE mor frgoian forthe ity rom anton pan fv Anlemettal si simply conan precigested ood anc wud ot ad total malin ‘The safety ofthe -aminosalyic acid (-ASA) drugs pregnancy isbest supported the cata on Salazopyrin which have been avaable for thelongest. Which of the following drugsis an inhibitor of cytochrome P450 hepatic enzymes? (Please sek ct 1 option) © Carbamazepine © Griseofulvin Omeprazole Phenytoin oO Rifampicin itl hil fe ns | os ----) sexo ETE | > tameicin Explanation ‘The correct answers omeprazole ‘The remainder of the listed options are all cytochrome P4S0 inducers. ‘A 45-year-old female develops profuse watery diarrhoea with lower abdominal pain seven days after undergoing laparoscopic cholecystectomy, What is the most likely diagnosis? (Pie ct 1 option) © Abdominal sepsis © Bile acid diarrhoea O Campylobacter gastroenteritis © Pseudomembrancus colitis © Pseudo-obstnuction Abdominal sepsis ie seid cermoea © Cmpbcostoeneiis ry © Poudomenoonauscoltis CT Pseudo-obstruction Explanation Prophylactic antiotics ae frequently given in both laparoscopic and open cholecystectomy. ‘Typically broad spectrum antibiotics are administered with a consequent ik of pseudomembranous colitis However, itrmust ‘also be remembersd that Clostridium difcile may also be contracted on the ward. ie acid dlarhoea may affect 1026 of patients folowing cholecystectorny, Typicalyit is postprandial the bile, with no gallbladder to store itis excreted directly into the gut A33-year-old man was admitted to the hospital with acute bloody diarrhoea. He had had blood in stool occasionally over the last 6 months but he had not visited any doctor forit This time, he presented with fever and tachycardia. After admission, an emergency colonoscopy showed inflamed mucosa with occasional bleeding spots. A biopsy was taken fromthe colonic mucosa which revealed inflammatory cellular infiltration only in the mucosa with aggregation of neutrophils in crypts. What is the frst line of management in this patient? (Please select 1 optio ‘© Aminosalicylate enema © Intravenous corticosteroids Oral sminosalicylates © Oral ASA plus oral steroids © Oral steroids SseReDeleeE TEP HON ‘Aminosaiyote enema Ineaverousconicosersids SESE (fal aminoslovates © Casipusccteods mT rat stern Explanation “This patient has MESES (UC. There are typical crea features and also, mucosal biopsy shows pt abscesses. ‘cording to Tueove and ts ite, ths patent nas severe UC. Thus, according othe NICE guidelines, ravenous comteosteroids are used toinduce remasion Topicalaminosaclateis used in mito moderate UC especialy thatimted to rectum or sigmoid clon (Fai ASAis so used in mito moderate UC, according tothe patients preferences although taloneislsseffactve than ‘ical preparation In infammatory exacerbation, igh dose ora ASA induction maybe used (Fal ASA plusoralstrcidsis used in acute exacerbation of mid to moderate UC olf sded UC. (ralsteroids are usual aden in steo-2 therapy. may be used asfiline i theveisintclerance to oral ASA, rermsntnent einine esnthean in eee van ree icterslovegonn scone TEES Exacerbation of Cros disease Functional darthoes Ponies serio ‘Short bowel syndrome Explanation ‘symptoms of boating, abdominal distension and darthoea are very consistent with adlagnosis of bacterial overgrowth _gyndrome. The hydrogen breath test furthersupports the diagnosis. ‘The fact thatthe ESRis norma santicantly reduces the likelinoog thet ths isan exacerbation of Cros. ‘The raised MCV is elated to Byz deficiency, which is also caused by bacterial overgronth Both metronidazole and tetracyclines are used in the management ofthe condition. A17-year-old student retums from a backpacking trip to Nepal with a two-week history of offensive diarrhoea and weight loss. Which is the most likely infective organism? (Plo i option) © Escherichia coli0157 © Giardia intestinalis(G lamblia) Salmonella typhi © Shigella flexneri Yersinia enterocolitica Stenecins TTT er Sie ores en) Explanation “They aries ira captef vers mes NN te sty gre Gira ome otc inca cause overs does is vanadyl contaminates ate oretrean poole Te nabston prelstnee dass theweAsand eso caps verano Dares tobe the pesning seta sa hi les el Tsien fled yen ‘soreasanitalpo,ardug Th dons a becine pests ldtomabbapen ances Uke ‘teers enone demos, Gide es0/ secs nmnccorostere nxs5 nes mance osisciroreyvaalona-ytsineeslertoptoste nemo! bvemicebepyistesed wht ‘sine cose meranazte cas Metin as ben show cie cv OKeT bes ance be ‘oiees an tncio Promonyen cate es pene meena pence Resnar eto can crea coH O17 arcane ot inctus Bao uly ass ey daroeawen esas nak ean complesactyheuetcuranie sje Sigler Yeas tly ne ert _somonatayoncatesphod ver ten Ypealy pastas asemilines wth eine saree ‘22-year-old man retuned from a back-packing holiday three weeks ago. While abroad he developed bloody diarrhoea with, abdominal pain Stool cultures have confirmed Salmonella enteritidis: Which of the following antibiotics would be firstline treatment? le t on) © Ampicillin © Ciproftoxacin © Erythromyein © Metronidazole Tetracycline eee ee | pete Coots TT © eens SII [6 | its ae Explanation Creflomacinis the asbosc af choice forthe WeatmenttSalmanela- S03 mgbafor 10-14 days Dianthoes occurs due to increased waterin te stool. The defnitonof chronic darhaea ste abnormal passage of thee or ‘morons o liquid stots per day for mare than four weeks and/or a daly stool volume » 200 m/day weight > 200 g/l). Amen or cprolxacn canbe usedor de tueatment of Shae Enytomycin susein Campylobacter unt etronidnaoeis used for Costu die. Tetracjineis given for Yersn enrerocotice Which of the following is consistent with a diegnosis of insulinoma? (Please select 1 option) ° o High festing glucose, low insulin, high C peptide Low fasting glucose, high insulin, high C peptide Low fasting glucose, high insulin, low € peptide Low fasting glucose, low insulin, high C peptide Low fasting glucose, low insulin, low C peptide High fasting glucose, low insulin, high C peptide. Low sting gucose high insulin. high Cpeptise EATS © Low fasting glucose, high insulin, lew Cpeptide hencocoide EE ) Low fasting glscose, low insulin, low C peptide © Low fasting glucose, low insu Explanation In patients with aninsulinoma there is low fasting glucose due to high levels of insulin and C peptide is elevated. Insuinoma is a ancteatic endocrine cell tumour. Patients suffer from recurrent hypoglycaemia due to the secretion of insulin. In contrast, insulin overdose will cause high insulin levels but low C peptide, A35-year-old woman with alcoholic cirrhosis is admitted with deteriorating encephalopathy and abdominal discomfort An ascitic tap revealed a polymorphonuclear cell count of 350 cells per mm? Which of the following is the most appropriate therapy? Please select option! © Intravenous amoxiciliin Intravenous cefotaxime © Intravenous metronidazole © Oralneomycin © Oralnorfloxacin ‘(Please select 1 option) Intovenoue aii rocco IID a © cnt TS eS Explanation Ti a nas SPONSE PEGE 2s sugested by nepal sory, asc nd alec pajmerphonuceaeaunt wenn measetctap Is mastcommony seen in alcool BREE andthe causative rpaisms ar usualy Escher col Keb, pnsumoniscr Eoercoce (Compare this wht mised grt enn other forms of partont Sencng some atc idinblood cutive betes mcrae the yield Ina reatments wth bead spectrum antbiotis such ascefotsime Noonan eracammende ershar erm prophy Which of the following features, seen on barium studies, is typical of both ulcerative colitis and Crohn's disease? Pre ject 1 option © Cobblestone mucosa © Pseudopolyps ° Rose-thom ulcers ‘Skip lesions OQ Strietures RES aE MONEE eee reee © Cobblestone mucosa - reopois CEIWaER | © Rosesthorm uleers © | Skip tasions en...) Explanation Pseucopolyps are seen in both llGsfativé Gol and Crohn's disease. Each of the remaining options listed tend to be features of Crohn's disease rather than of ulcerative colitis. The criteria for the clinical diagnosis of toxic megacolon includes evidence of colonic dilatation on xray in addition to which of the following? tt O Electrolyte disturbances © Heart rate above 90 beats per minute © Hypertension ° Polycythaemia © Pyrexia above 40°C eons: Henan ee] Exponaton rcrsteomutoneiene tect sstentcresoumectoe dason lth aro oe ees ‘airasr een canst bart digg noe eg tan stra Geman etsemenagnetin inet sertstetnng cmt aprovtieemegen Fntguonc nic les esenp eaten seen 1 seoepi coon 05109766 1 een 1 serie courant hens |Aeb:yaoldmanwitn iawn cone ver asease acon ts aeohals emitted to hospital contin, He cent alang sptonoactone 400 mgand roremise4Omginaddtontoaneraintaminauplemant ‘On wamination her ae peohe stigmata ofchoniclver rear and hing dunes can beefed. isputserate 102 beats er minute and blood pressure 95/40 me sosum emmov ware) Potecum 8mm asa Uren aemmo, es casting ar umavt, eon econo the Bitch Society of Gasticntarcogy uidetnes onthe manasmont of ascites whatithemost appropiate ay tomanapets rtiemanS yponataemia © Piaresret (© Nochange ncurentmanaserent © Stepdiretcs ‘© Stepdureicsand sve ners sing ‘Please select ¥ option) Nochanerinaentrangarnt Stop ress © Stop uses genera ine ED ‘Explanation aves win von nr dseae and atte fn devel HESS ne mnagument of hichcan be det ‘urevenapy fr themanagement of estes ofan corrbutest the ypenaaema. ‘The th Socio Castoentrclgy genes suggest that whee the serum zaumie 120 mmoV- dere hep ‘aude topped ang pate shaulérceeolume xan wth older aera, ‘These guns abo aise hat iret shelton beusedinpatint who areca eucaemi net on ures an ve severe ase wi roxmal seu crearne espe ntaventon ener tan atu mentong sted whee me serunsoaunis 126-135 mmeVL_nte gs {21-125mmul/L where the sum crexinneisnema re apy maybe contrued Day meet Berecuced wth ewe Hopping traces 1 ie sum isintharanget he seu cresting dec shoul stopped an ptt shal receive volume Which of the following demonstrates autosomal co-dominant inheritance? (Please sel © Alpha-1-antitrypsin deficiency Cowden’s disease ° © Familial adenomatous polyposis © Hereditary haemormhagic telangiectasia ‘O PeutzJeghers syndrome Aporaniesn dcncy eee oe) Pes eahessnaume Explanation ph rts AYA dency inan teams ce-doinat ore -both es connate phate. The mst conmenallec norma whist tere are 10 ara alles Beading deceaed AATIe he most. ‘armen ae Zanes nail aig naa ray hae eons mas of ATAT DAL pode sua Potent pent delopmentatedorte pena he anh sates iets te conto steam recs inte ass state mere pac elultarspon fans anism eaangeSeaumuston wei ee seanepate ny Concer case an axon dressing nut havatomasofskinandrucousmentranes Fant atenaratus ayes he commonest senor oypasssyarne demonstrating sso dominat eran naronnage eng Ose Hebe Renty sere demons ats ornate anais razeredtytingectans ecg tsk and mucous emirnes set ptetna hearse oc surnusyin demenstatesusonal gmat iertance Thre chron mucnetnonis Pension pops canorcaranerein hepatic bt atecommon intel A26-year-old presents in the first trimester of her first pregnancy (six weeks gestation) for an antenatal check; she feels well. Blood tests show a bilirubin of 40 umo//L (1-22). The other LFTs are completely normal Which of the following is the most likely diagnosis? (Please select 1 option) © Cholestasis of pregnancy © DubinJonnson syncrome (01S) © Gilbert's syndrome © Primary biliary cirrhosis (PBC) © Primary sclerosing cholangitis (PSC) |o)eeareees Oo | Dubin-Johnssan syncirorme (04S). cet ncone LETT © Pama bin ciinoss eS as Explanation Gibertsis the most common condition causing mid isolate hyperbiirubinaemia, PBC and FSC are much fess common conditions ane! are almost aways associated with rise inthe other ner function tests, -particulanlyakaline phosphatase ALP) and gammaglstamytrensfrase GED. 8s much ess common than Gibert’s. ‘rtahepatic SIEBER) relatively commonbut usually accurs inthe secondor thd master ALP usualy high iskinereases with muta. secre == neat 0m oor vcoeon retest ‘Explanation ‘ontrmaion ofr panceatcinsficenyrequres a functional tes. anstaralasesthebestopton One wot _soolregured fran anattlevelofelanaoe | canindeate + sssomee/e- sete pacseacnauseny ‘+ r0n200 me/g-roseteinsuoency 1 -2n0mee/e- nema “Theadvartage of ths vers tter oponsistat hetet ean bepearmedens sre pefaees ome ‘Abdominal CT and MRC may buen demonstrating paneret anatomy neusng ress of caleestin and Foss ‘consent tO they are rot functional messi Faecal ats genay aoe now because or omelette patentn otectng a tera, yatogen breath testing sea the cagnosso acetal egewth sync A 72-year-old man is discharged from hospital fallowing a stroke. During his stay he was started on several new medications. He presents with diarrhoea. Which of the following medications is most likely to be the cause? (Please select 1 option) © Clopidogrel © Enalapiil © Metformin © Pioglitazone © Simvastatin ‘select 1 option) Clopidoerel Enalapril Metformin Pioglitazone Simvastatin Key Learning Points Gastroenterology, Pharmacology. Therapeutics «Metformin can commonly cause gastrointestinal disturbances, Explanation Although all the medications listed could cause gastrointestinal disturbances it is metformin that is by far the most likely. A52-year-old woman presented with history of worsening dysphagia over many years Recentiy there had been episodes of il- defined central chest discomfort and nocturnal cough Which of the followingis the most likely diagnosis? option! Achalasia Barrett's oesophagus Motor neurone disease © Desaphageal carcinoma Pharyngeal pouch PEERS Sets TORENT a] a naa © | Meerncrnadanse (esophageal earcinama Pronrecclvoucn Explanation: ES presets restftan inte tind ith decades ‘Syrmptomsusually develop years before the ptint present Vague chest scomfortiscommon Thiny parcenthavea racturnalcough due to asiation of a2sophageal contents ares cescphagus oes nt cause apna MND causes dspasia Ge toprob¥ems wth chewing andintatinga wallow and woud not cause ches discomfort. ‘esophagestcacinomals vy untikely duet the curtion of symptoms ear) ‘A pharyngeal pouch usualy presents inthe sth to seventh decades wih regutaton and wou not cause chest dacomvort Which of the following is an inhibitor of gastric acid secretion? Pleas lect 1 option) Q Acetylcholine © | Histamine © Prostaglandins © Stomach distension Swallowing eee ee Acetylcholine Histamine © Prostaglandins ED | Stomach distension Swallowing, Explanation ‘The correct answer is prostaglandins ‘The remaining listed options all stimulate the release of gastiic acid. A 51-year-old male labourer presents with a haematemesis and undergoes urgent upper Gl endoscopy. Using the Rockall score, which one of the following features would categorise him into a high risk group for a subject presenting with Gl bleed? Please select 1 option! © Abdlood pressure of 134/88 mmHg © Anistory of ischaemic heart disease © Aplasma glucose of 7.2 mmol/L © Apulse of 90 beats per minute O Hisage ‘atraon ‘reese snetanin cease pest, + And Aman is admitted with acute abdominal pain and vomiting He is diagnosed and treated for acute pancreatitis. Which of the following features is associated with a worse prognosis in acute pancreatitis? (Please sele ion) Plasma glucose of 11.1 mmol/L (3.5-5.5) © Plasma sodium of 125 mmol/L (133-144) OQ Serum amylase of 1200 IU/L (24-100) Q The patient is SOyeers of age O White cell count of 13.9 x107/L ae Pomerat) I Seman 20 10 omen ST partion ‘Thee ume tren sacar pe nn ees pop eset acaNe fansite en admin tat siamese prs eae 1 baesssyeusol- pont weno pot ‘in m-th rere preset tous + Hensoentat ot Osos 190r¢ 1 Uearseat rm ore esse eit o2-woreg poe 1 Iuedetersemee- 1p + Msseveeaton-0m pont Which of the following stimulates bicarbanate secretion from the pancreas and liver? (Please select 1 option) Cholecystokinin (CCK) © | Gastrin O — Motilin © Secretin © Vasoactive intestinal peptide (VIP) Rass aeRE EAD Cralesyrsenn COD > | casein © masta © secetn © Verosctveintertnat peptide WE) Explanation ‘Seretin secrete rom the smal ntestine when there f acid inthe smalintestine tints gastric motlty and acid ‘roaution ana stints biarbonatesereton Tom the panereas ana Wer (CCK stimulates galbiader emotyng ad pancreatic eneyme lease astinstimdates gastric acid secretion and patie motity Mottin simultes intestinal penises. ‘iP induces smcoth mute elsation, stimulates eration of waterinte panera juice andi, ard eausasntbon of ‘gastric acid scrtion wht inthe intestine it gretly stimulates secretion of ater and elecrayes. A 28-year-old lady develops abdominal pain, jaundice and ascites worsening over aweek She drinks ten units of alcohol each week and takes the oral contraceptive pill Which of the following findings would make a diagnosis of hepatic vein thrombosis (Budd-Chiari syndrome [BCS)) most likely? option) © Acute liver failure © Alanine aminotransferase (A) of 345 U/L (5-35) © Ankleoedema © Ascites fluid protein of 38 g/L © Tenderenlarged liver Ne eae eine srirctansferme(ALT of SUA. -39) oteacedeme | ‘Ascoeotiud poten ot 38 @/L renceresangciner ED Explanation “remote cas of Sn acitesever Iernyin young cana: + vss ncn SSD vis HA, HS Vs EVD + cvs ool tamale) + aldommuneneais + hepntewinworbrsohen rected by pean ecrlcntacetve iO “Tropretenc ot ner ere an cedema anda nue ees donately eens between these aetleien ‘TheALTof34 ismorately erate and compatible wih 6. Wh valor druprestedhepats thepeakALTisusaty _nuch ger tan hth ALT mayaleay be on the way down she hashed symptoms foaweek, ‘Tender hepstrogay sone ofthe haart ECS. Inacute severe wal ausimmune or dus/tosn etd ver dene the recrtcverdeeaees nee. A 37-year-old homeless gentleman with a history of alcohal excess presents to hospital with progressive abdominal distension He now complains of early satiety and abdominal discomfort as a result of the distension, Examination reveals significantly distended abdomen with shifting dullness. An ascitic tap is performed and the fluid sent for analysis According to the British Society of Gastroenterology guidelines on the management of ascites, whatis the most appropriate first-line treatment for his ascites? fect 1 option) © Amiloride Dietary saltrestriction © Furosemide Paracentesis © Spironolactone eee Daron 2 cent TS = Explanation “Thetoutmerto eee age,symptomatcasetas sage vote ape pracete Where tnevaumecfasctes not sfcinttowsant aacetasshenfis ine tatmenta eisai one ‘mor than90 mei sndspeonoctone. ‘nal ose ot peonlctanentiseting 100 me/sayand maybe ated upto $00 mg/day Once De maxim dose sproel:tone has een ened fuvserde can eae there nacant asco ceunulton nae el ‘unctonandelctryes witout cues ose arcu stead saa g/g Upto 160, mg/eay2stoerateaer needed, Furosemide has poor eicacyin cos -aeie not genetay recommended oruse i iS nase a5, troup pradces anatrues seesye Ssentexty neo speonalctne ‘ume ay be used stadt roomie A25-year-old man with along history of heavy alcahol intake is admitved with nausea and frequent vomiting, four hours after a meal ina restaurant. During review in the Emergency Department, he vomits « cupful of blood, Which if the following is the cause of his haematemesis? ase select 1 option) © Duodenal ulceration © Hemorrhagic gastritis O Mallory-Weiss tear © Oesophageal varices © Oesophagitis a eee | © duodenal lc-aton Haemosrhagie pestis | 6 eng tot ET © cmcpnaccsherees AAA 2D Oesophagitis Explanation, ‘Persistent vomiting can eventually lead to small tears in the oesophagus, leading to the vomiting of ed blood. Varices would produce large volumes of blood (much more than just a cupfud. |A7S-year-old patient presents with watery diarthoes, Hes passing large volumes of watery diamhoes, approximately tres a day. with no noticeable blood. thas been present for approsimatalyfve months andi gradually becoming more frequent often wakes him at night with the urge to defecate. Liver funetion tests calcium and urea and electrolytes are normal, Stool microscopy and culture are normal, and Clostridium Aiffcile toxinis negative. ‘flexible sigmoidoscopy organised and the investigator reports to you that thelarge bewel appearsnarmal Biopsies demonstrate mild thickening of the subepithelial collagen band with 10 intraepithelial ymphocytes per 100 epithelial cells: Crypt architecture is normal without evidence of eryptitis From whieh ofthe following treatments may this patient benefit? (Please select 1 option’ Gluten fee diet Lowsesidue diet ralpudesoniae ° (ral cholestyramine (ral prednisolone ptraton nbesbesinfeen anh oni tr money nn pss hey abemeeoess seman ean oes suntan meester wenden tcoxecalis mote dares chnoxan otracnd Sep encae mone enaenen ‘enedoesantug near ip esse tsa dpe Mace ‘ohmostanne nite xh pensungenecsemacy neni anhes niche ‘Rewer socima gene ye tens noneneneaneney esata nang na tenement Re Sseyobemetecoe int cotati “ee ecitehgcente seater teesiesmateron Ai smecnigecaatan noe s pptansasstn cps Bere) ons ‘Ceetyanea conden bas thoes he een cnpra bese po es nmap is negro ies eye an ts re ‘A'56-year-old man fromm Thailand presented with abdominal pain and a mass in the right upper quadrant. He reported that he had been diagnosed with viral hepatitis several years previously and had several treatment which he states reduced his disease burden significantly but heis still a chronic carrier. He reports no high rk sexual behaviour or Intravenous crug use. Investigations showed: Serum alpha-fetoprotein 13,5001U/L «10 \Which of the following isthe most ikely underlying viral infection? (Please select toption) Hepatitis Avirus Hepatitis B virus © Hepatitis vis Hepatitis D virus © Hepatitis virus O RepatitisAvirus Hepatitis virus ) Hepatitis Cvirus Hepatitis D virus == © exttse ons Se Explanation ‘The patient has chronic viral hepatitis and presents with an AFP elevated to such a dearee that tis essentially Giagnostic for ‘hepatoma. The underlying cause is either HBV or HCV. Since his country of origin isthe only other deta given here this gives a clue to the cause of his hepatitis. There isa higher ‘prevalence of HBV in the Far East, consequently, the most likely viel agent is HBV. Arh maldsaan comer tothe trenton rc frevem Sa renee th amc ‘roe onn emt rorraror Gers eutceorsiedt aor areola ‘sana at parecunmgusapine cour tae nape lk tne cheese Romany noms sours =n te ccm aan Panter aos, ce ama ‘Vanaek wtp, croton tteaby stem abet see? ao 2 Cmte { : s ‘Please sefect 1 option) pasceanmr cy TT © conn ES Siamoisescony Explanation ‘Theatsence loner Gaymptoms asesthe posit ofa gh sed oon sen resutngntemicoctic anaemia ten ‘ere Te best nyt recut an undeygcarcnoma wits cocnoseopy er adecuate bowel rapa Age pectic sant serapecticncidence ater forthe UK(2009-201) focal cance sage he apposmates 0355p yeatathe 70- “ayer ae group Fr tisreasen. fhe investgationisessnba especialy pen postive faealorcut Hood FOB samples. ‘Siansidcscopywinot éequta visuals the whe olen, abu enamine 3 senstive for smal pohp=versus ‘enkrascopy although batum enema wel be analemtve optionifclonaseop a tole _cabdman itn contrast asthe of ising mal ight siden but T colons lbichincude fl ‘bol reparatn)inaressonable emai in those unable fa tote clnoscopy opsicendoseop is generaly masseter magag he sal bowel wheetha animpacanrobia sting anpceysasa Which of the following allows a diagnosis of spontaneous bactenal peritonitis (SBP) on ascitic fluid? Please s tt n) © Lymphocyte count 2100 cells/mm? Lymphocyte count 2250 cells/mm? © Lymphocyte count 2300 cells/mm? © Neutrophil count 2100 cells/mm? © Neutrophil count 2250 cells/mm? ‘hich ofthe folowing allows a ignoss of spontaneous acter pentonitis(SBP on ascii Fuk? (Pease steep) 5 ymnacyte count 2100 |e | amet? (© emerson’ Jo | sawn enn? © Neutophicounts2s0ce6/mm? Explanation ‘SeP isdagnesegt teres + hgh ate Pui neutrophil count 250 cee? ‘+ postve act tia bacterial culture and ‘+ sbrence of secondary cates of peritonitis sc as bowel paforaten. “The absoluteneutephicountn ascii Muti alte’ by mutipin the total white loo ell count bythe percestae of pausophis inte atesenti, ‘A.30-year-old woman prasente with jaundice and her investigations reveal: Haemogiobin| 0a/ 15-165) Reticulocyte count 10 «109A. 05.85) Serum bilrubin Sou a2 Herbloog finn reveals the presence of spherocytes Which ofthe followingis the next most useful investigation? ‘Please select 1 option) © Abdominal utrasound scan © Drectantigiobuiin test © Glucose-6-phosphate denysrogenace actvty © Haemoglobin electrophoresis O Redcellosmoticraaiity Sees Adorinatutrasound scan ovectngcbin EEE 1 9 Redcetosncictasity Ee Explanation ‘The ests gon indicate SRA RRRR ot which sphrocyts are typical and given the age the patient themost met eause immune ‘The rststepn analysis ofa spherocytichemalyticanaemiaisto determine whether the process shemale ornot The bast ay to dots 2 éectarigbutn test negative. one cold go onto cont HS with other ests, ‘The osmotic agit tests urvlable and sno longerecommendedinrotinecncalpratice. Osmotic gradient _ktacytometys used to citferentat Bsa spteeys rom heredtary stomatocyoss, buts only avaiable) “specialsd borates. the agnosis is equivocal the cryheemoysistest and EMA binding can be used Inatypeatcaes gl electrophoresis analysis of enthacyte membranes the test of hoe. A yearottmalepresets witha twomont history of depots west ossane genealogy Hemedial story Includes 2 prevousieseanampaess ae tsrek far when neta spn anépetingopi ‘camino oval resiualiet sided hemiparesis togetherwih apaeandsighty auniced appearance. Investigations show Hoamegtia sogt, 30180) ey rH 20:96) white cat count a0, em Protas asa, 30-40) nna showed inreeed uoblinogen Which ofthe flowing the ne most peropriate investigation? © Bore arom aerate Direct antbutn test © Entosccpy © Serumnapegiatin © WtarinBraconcentaion Se © Bone marow aspirate Drectantiecbuintest fdoxcony Serum haptogibins © Varin 8:2concenvation EEE Explanation In his situason serum Bj esimation isthe corectchaice With a pancytopenic picture and raised mean corpuscular volume MO, the most propriate step isto check the Brae fle. ‘The other choices are considered onl ater thebaic assays Hoemoiyss doesnot explain thelow WCC, nr the thremboeytoperia -Anaptoalobinonly ads weit to cagnosis of haemsysis, nd an RBC-abelled scan would add reser sensitivity othe “dagnosisothaemoysis ‘Themisjunccitpcalofmesclatc rae tain on) becaee tnceted ston ore6 zante ba, A 45-year-old woman presents with pruritus (On examination she has clubbing, palmar erythema and spider naev. There is also evidence of exconiations and xenthelasma. Blood results demonstrate deranged liver function tests with a predominantly cholestatic picture but the abdominal ultrasound scan is normal. A subsequent autoimmune screen is positive for antimitochoncrial antibodies. Given the likely diagnosis, which of the following HLA antigens is associated with this disease? (lease select 1 option) OQ HLA OQ HLABS © HLAB27 O HLAB35 O HLADRE HUAAR © | piven > | muaner © Hes Re oe TS Explanation Tris woman has erimary bilan ith PEO nen ie aseocatc wit HLA OR Antmitachoni antibodies are postvein 95% of cases of PEC ‘HLA-AS is associated with haemochramatosis, HLABS is associated with Behcet's disease Subacute thyroiditis hasan association with iLA-835, GED SOMA is associated with HLA-B27 ‘An 80-year-old woman preserits with confusicn associated with a chest infection ‘She received standard treatment and four days afterwards she developed green, liquid diarthoea which shortly afterwards contained some fresh blood Which of the following organisms is most likely to be responsible for her diarrhoea? (Please select 1 option) © Campylobacter jejuni © Clostridium dirficiie O Escherichia coli0157 Methicillin-resistant Staphylococcus aureus © Vancomycin-resistant Enterococcus (0 Campytobacter jejuni ° crimence Ty © cxrechocoto:s7 my O. | Methicilin-reststant Staphyfoceccus aureus _-Yancomyein-resistant Enterococcus Explanation ‘This s typical of Clostridium infection with pseudomembranous colitis induced by prior treatment with broad-spectrum antibiotics such as cefuroxime, Augmentin and the macrolides. Itis treated with oral vancomycin/metronidazole Which of the following is true of Spontaneous bacterial peritonitis? option) © Asurvival rate of over 50% is expected at one year © Gentamicin is the treatment of choice © Ischaracteristically caused by anaerobic bacteria © Isdiagnosed by culture end examination of ascitic fluid © Isdue to intestinal perforation ‘(Please select 1 option) cuniarat of vers eorpacted atone eat Cenczicin isthe treatment of ceice ‘schractersicaty caused by anaerobic txts © Nedagrosed bycutweanderrination ofasctic nus IE severe estinapetration Explanation ‘eh sa faquentcompieatonfthe aces of thoi his agnosey asc id examinaton which reveals a PAN count £f>250/m) SP haspoer prognostic siniicance wth gone y" sv tera agnosis of etween 30-50% is. athe nat sugges aspntaneousavant at snotaconequance ofits pttorton Is speculte ha theiectie organs may ek int the xc vi the blood Fo intestinal verronth ‘Organisms sneuizne cuturedy act collecting nto bled cute ots Is Wpcaly caused by aerode gam-egathe ‘bctria Hence antbetes such asco amie, zosen or cprotoacin are play sedate ne weatmert cava Nebo conser aerobiconsnsins Thay am nae pecs dines aculat anise nas ‘osu. thy reproduce bestnaerotic condisonsbutcanalo reproduc in anaerobic condone blgatesncerobeslee ‘lst ae ile ty he presence of onsen ss cpposedo serotcerantnaerobeslea Lactobyciua which ave ‘xcs frente aarti] tabla at sent ta the presancact cay, Aycan cones me gaat te ee cota Ov eb on ae Ma saree ann one Parr pas Ow Sea vntcine Panay ayaa © Pimaysciwcsingchoinens Explanation "hsmanshior ota antenna wih aloe pete constant hobs her dese Yn Inti ptween 75 and So patents th pay ss chong FSO aero havc ietrmatay ove dears, though on acur 4% estes win anton bowel dese develop the condton There oenanincesinigMandsaNCh CLatbodessaANAate reset pt 8% and kf patnt, especie Enesco revogaéechlrloanctetonmphy (ERCP. he god sancsvesteaton, demote utente inne Iachi em deigh REE RE enoungopanreatogrpry MRCP sate peremec nay. cis ene pregesewih edn neo nearest ataroun 2 years Patents are adctnalyatnteasdrk A 48-year-old man presents with haematemesis and melaena, He admits to high alcohol intake. ‘On examination he is shocked, his heart rate is 110 beats per minute and blood pressure is 92/74 mmHg. There is evidence of leukonychia and abdorninal examination reveals tenderness in the epigastrium. Whats the most likely underlying cause of the gastrointestinal haemorrhage? (Please sele 1 option) © Gastric antral vascular ectasia (GAVE) © Gastro-oesaphageal varices O Mallory-Weiss tear Q Peptic ulceration © Portal hypertensive gastropathy pases dl © atc an scree GAVE) Casto cceophogea ees © Matar easter © Pepteutecton ES > Poca morensve gatropathy Explanation Pept cet he commonest cause acute upper gastontstil Gf aemorage Evigasirc tenderness poms tonars peptic cer tee The histo ofhigh accel nthe cn be miseadingin decthe thecanict torts vaca. Incase the ase the mos common cus of an aeute upper lecwneh pete etalon 25-50% ofease ot asta vascular relatively uncon cnc typical preva cute ofton-ieny ane «ares fehone Gl eoalos Overt yma ef upper Ginaemenhage mayb pressed by amination eam eet ager ascoaguats Temean age of dagross arcund TO andthe an asacaton win seeder Portal pees gsropttyisapcory ucestood cemplcatonof poral ysetensionsualyasarestfchrene ner SseseThoeisatypcal mosaco sakeskn Meappeaarce othe asc rucoss Mc, th neeasasveiV fhe ‘conden can cevop ase ectais ncbecome vey rable Typical resection wth cron Geo oss bu acute ‘nsemerthagemay occur A40-year-old mani referred with gastro-oesophageal reflux disease (GORD). Which of the following concerning GORD is correct? (Please sslect 1 option) Acid suppressant therapy should not be given continuously © Endoscopy is mandatory © Inthe presence of Barrett's oesophagus, the risk of future malignancy can be assessed endoscopically without biopsy © Oesophazeal pH monitoring isa good guide te therapy © Symptoms do not correlate with mucosal status at endoscopy ‘Act suppressant nerapy shoud not be given contnuously Endoscopy ismandatory lino presence of Barrets oesophagus he isk of tutu malignancy ban be assessed endoscopically witout Dionsy ‘SescphagealpH monitoringisa good guide to therapy Sptoms donotconeate wth mecosal sats atencoscony EEE Explanation Symptoms GORD da not correlate withthe mucesal appearances at endoscopy, ‘though endoscopy should bs perfarmetin cases that are not clear-cut of donot respond taproton pumpintbitors PP it Is notmandatoy ‘Thersk ih Brot and hence the elagnosis of Bares can only be clarified with biopy. Monitoring of pHis nota good guide to therapy bu symptomatic improvement isa good gue tothe efcacy of theapy PPiccan be gven continuously wnerethe cagnesishas been satisfactorily proven and elapse of symptoms persist er thera ‘A 21-year-old woman is referred from the Emergency Department with a paracetamol overdose. Which of the following is an indication for liver transplantation in acute liver failure as a result of paracetamol overdose? <1 option) © Mtterial pH=7.35 © Bilirubin >200 umoi/. 2 Creatinine >250 pmol/L. © Grade encephalopathy © Prothrombin time >100 seconds (INR >6.7) © ceive 25, Cael erst Protvenbatineoosecndnn <7) ER Explanation ‘Trekngs oe Crea ee ost mc aptecpognesc oops oes cE eI Fume tnese cranas srighspeccty tr moray athe stat ncrepstvepediie aan et) ‘antewiestopucr enue sbodte cee er ressrten Fo GME oa to ARI ce INRo€5 1 =100meconc ands sine» 30)yel/ L634 g/t nga or tet ance. tte 2diys ay to 7 days ee) Oto 14 days Explanation “To maintain gu bare function and prevent early bacteria rancocation, enteral feeding shouldideallybe started within the {ist 24 hours of admission The presence of fuid collections in severe pancreatitis raised pancreatic en2ymes isnot necessatly a contraindication to ora or enteral feeding. Ina subgroup of patients there is correlation of pain, recurence of csease, or worsening of ud collections wth feecine. nether ora or enteral. the fuidcolections are not suitable for crsinage orf enteral feeding i not achieves within 48 to 72 ‘hours, supplemental parenteral auton shoulé be provided A 35-year-old man is referred to the gastroenterology clinic with persistent pain in the rectum. The patient reports recurrent. painful oral ulcers. On direct questioning he admits to red and painful eyes. He also reports he has recently noticed an ulcer overhis scrotum, (On examination there is evidence of ora} aphthous ulceration; abdominal examination reveals tenderness in the tight iliac fossa and a bruit on auscultation of the abdomen. Rectal examination demonstrates perianal ulceration. Whats the most likely diagnosis? (Please select 1 option) © Behget's syndrome O Crohn's disease HIVinfection syphilis © Tuberculosis eens ITS Explanation erate nemesis fanhonnpsterhsony ‘ns or aie ipealy ne dandtout sais Rismoreceninmen ithe Mage Ean wemen nha Dros tes note IE which ae pa ecuenanc onsen and Wot | * pursuant sents ane * enngsercepte na ne eacsene tema tove dca icons eons eters em anaes. Donbeiducen cancer ne gatontstral ac but wemet comme unanthe eck rg ht ‘Snandeesphnit™ “The yesene fencer nd he sec iat hve proms at gaping nee aera erence boa Dut whe sate foarg vacate? ‘A0-year-cld woman wth along histor of alcohol abuses prescribed phenyin fr eplepsy Examination wasnormal axaptfor sliver edge Hertubtood countreveats Haemogbin 100e/ cv van White cat count 2210/1 Pratletcount asniorA, Wiichis the mot key explanation for these resus? (Please select option © Neohoiciver sass © rolstie anaemia © Foesciddfcieney © Wypotnyotasm © Vasmineaetsiney caissies (0-96) a (450-4003 0) | Acohotic wer diaassa Aplastic anaemia © folcacidcetceny TI © | Rypothyroigism ee no) Explanation Folicacd deficiency would give al these results. In addition, she has good reason tobe folate decent since she inks a considerable amount and taking anticonvulsants. Ac urges on its own would not make her leucopenic. Hypothyreigism would cause a raised MCV, but nt the other parameters Seuny doesnot cause his pcture ‘Aplastic anaemia coulé cause this haematological picture bu the clinical scenario leads towards foe acid deficiency. 38 eaelamanainoants kre fn per iy eters ergy Depa cl ‘seoniaipan Hetseu mate hs sued fominceitg teams big eaten. on enenien hi P105/7 eel Dana ‘ters bowl aomis nate He penis aenege| eset tect nano, ras seo" Nesropie/ mines mo petoacancracomyen Emonyenandepotoncn Exthomyensndmerensne yeni. a sdoman eel nde bt Set nS Cpmtoscnandenconycn rey (Covemoricae Erythromycin and ciprofxacin Exythromyein and metronidazole © Pomnsinand taco Explanation ‘Te diagnosis here is spontaneous bacterial pertenitis, Note the history of alcohol exces, raduly increasing abcorinalgth, and slevated neuropil count in the ascitic Mule SSUES NER onutd be managed ages, and whist piperaciin and arobacam te an ppropste choice in those who are not penicillin allergic. In this case ciprofloxacin and vancomycin are the defauit choice due to the presence of eel alta. Orel continuation therapy is witciprofonacin lane of co uimetazoe. Co-amoscav snot appropriate due tothe presence of paniilin aller. nor the pjpeacilin and tazobactam cortination. The to exytromycin combinations do not provide en adequate brescth of coverage ‘32. yea-of female presents wih prurtus and junlce Shes: 30 weeks estatonn her est pregnancy “woveeks enter netadbeen creates tytn ENT surgeons tter presenting tothe Emergency Deparment wth inrctshle nose leeds Liver function ests eve Aq UA. 6-40) ‘Akai phosphatase out con fiewn Syme one Serumbie see 10o8mesnamaltire Wien fthe folowing statementsiscomect concerning his pation? © ALPéoes ntincrease nanormal pregnancy (© Matemsthepatc iced fw does notineressein pragnancy © Treatment ootons include V Nace estene \orces ar iagnesticofhvercnease inpresnoncy (© Wealhepatisis tee diagnosis ‘ALP dass not nreaee in anormal oenancy Matemathipascbinedfowdcesnoticeasinnegnny EAI ) Treatment options ineuds WN aeatyystone \arices are diagnostic of Wer seat in presraney © Wainepactsismetiey cugnoss EAT Explanation “The disgnosis here intahepatc cholestasis wich presents with marked elevated serum ile acs choline It resents inthe second or third iment and ususly the alkane phosphatase ALF) is 7-10 times normal wih ised alanine ‘wanseminase ALD, xparat transamnase (AST and trun, Cardiac output an blood volume increase pregnancy but hepatic biood ow doesnot. “Treatment options inchice ursadeenytlori cia, cholestyramine, phenobarital andvamin Ko teat the coagulopathy ALP cisesin pregnancy but notothsextent The placenta isthe sour the ase ALP. ‘Viral hepatitis the commonest cause csundie im pregnancy but the elevated tile ids make this untkelythiscase A 44-year-old gentleman with dyspepsia associated with H. pylori was recently treated with triple therapy. He returns to your clinic for follow up to ensure the infection has been cleared. Which is the most appropriate test to confirm eradication? (Please s Endoscopic biopsy culture © H pylori antigen on blood sample Rapid urease test during endoscopy OQ Urea breath test Stool sample for H pylori antigen rdancope pay cane Halon anager on doaasamgie Rap waste curing endoscony = © resomansest ED Explanation ‘The gold stand ts erconfeingHocatucterpyleiradaten the reabenes Patents aerureasbeled th seatonizatape FH pin pesertinthestomech taunt ensyrezpits the utandissope abled atone = preset the paensesled rst tin 10-90) Tian be mesure iter snl armas spectrometry depending onthecatbanuetope wed Abaeine trast apleisakoreqed or he patent Paontsoheld hve stopped antbictsat ast four wether tothe est bens dene. ae pte pro thts atleattve Weck The ‘esthas hah sens 85-980 an hgh specie 980 for exnfingH poieradeaon ‘Therapies testrequesabiopey saree tenet OG) ands lowe sera. Seramantbogywilemuinpootiveafereeeatonanchas lone seneity aspect awe Cuma ot gs iopynasaignespeioy 00 Dies sree morn Riua mee cree we ‘estng abe sensivy nH py wnenresstunt onal pe Rey Stools isle specie andsensivthan the ueabeathtes, ASS-year-old male is admitted with vomiting, He has a long history of alcohol abuse, appears slightly jaundiced, ang is dishevelled and unkempt. He was started on an intravenous glucose infusion and diazepam and he symptomatically improved ‘One day later he became confused, developed vomiting and diplopia, and was unable to stand. What is the most ikaly diagnosis? 1 optior © Benzodiazepine intoxication Deliriumtremens © Hepatic encephalopathy © Subdural haematoma © Vitamin 8 deficiency © Benzodiazepine intaxcation © Dalim tremens © Hepaticencephalopatry SEAT) ‘Subduralhaematoma Venn ceceny IE Explanation This patientis manifesting signs of Wernicke's encephalopathy with confusion, oculomotor signs and ataxia affecting gst and stance. \ericke's encephalopathy is a medical emergency, requinng urgent intravenous thiamine, ‘The episode has been precipitated by ntavenaus dextrose administration which has exiaustedhis vitamin Breserves, hence B \veamins must be administered to ll alcoholic patents requiring dextrose, Which of the following symptoms is most commonly seen in patients with achalasia? Please select 1 option) © Aspiration © Dysphagia to liquids © Dysphagia to solids 3 Regurgitation © Retrosternal fullness a ee -spaton Dysphaga tous eee TT egupeaten Eqplanation Dysphagia she mest commansymotanin patents wth SENSES wth 91S presenting wh ayspaginta sos and USK. wth pagina gids. Ae cvs te ect os of bear neurones ine myn plas Tae ato the prcton of see ‘ntesina polypeptide nit ox, acinar nitrate responsi aboarmaower esophageal sphincter ‘suncton andtalretoelorinesponse to sawing ‘Themachaniams responibieforthe ss ofinititoy neurones isnot wel understood Pevaus tues suggest heredity, rnewodegeneratve, genetic nfectous andautornmune mecharisms kistought mest ikelyto because by Wieland uolmmunefactestexaingto the isto changes and damages he mentee pus. Reaurittion of unigested foe or sv as occursin patents ith SR ond reauraton maeadtoaspraton| (cceureg in 89 Fatentsnay ao sur tom asensation of aistral tues Tolownga mea A'S8-year-old female with type 2 diabetes is admitted with diarthoea and vomiting She hasnoticec small amounts of blood in her stoo's. The vomiting had commenced one day after a meal of chicken and chips The patient's type 2 diabetesis treated with diet alone, Stool cultures taken by the GP reveal Campylobacter jejuni. Which of the following is the most appropriate therapy? Pie se select 1 option) © Amoxicillin © | Cefaclor CO Wftuids ) Metronidazole Trimethoprim Americ cofactor ova TD “Timethoprim Explanation Campytobacteris leading cause ofdlahoea| illness, often eaused by ingestion of undercooked meat harbouring tne pathogen Its also amajorcause of travelers darthoes, “The use af antibiotic therapy forthe management of Campylobacterinfection in adults s controversial Antibiotic choice in ‘his infection is erytaromycin, atnough ciprofixacin and tetracyctine may also be appropriate However, appropriate uid replacement and anti-emetics ae inital indicated: most units advocate no antibiotic treatment. ‘A40-year-old single man retumee from holiday in Europe with mild bloody diarrhoea which had lasted for two weeks. He had lost 2.5 kgin weight, had occasional lower abdominal cramping discomfort and a painful swelling of his eft knee. Which is the most likely diagnosis? © Amoebiasis © Crohn's disease ° Cempylobacterinfection Gonococeal septicaemia © Ulcerative colitis ‘Amoeba moet © Gohnsdisease ee oo) © | Ucerative cots Explanation Campylobacterinfection is one ofthe commonest causes of inflammatory diarrhoea. Abdominal pain is often a prominent feature ofthe iliness frequently locaisng tothe rit ilac fossa Diarhoea may be mil or very severe often with passage of ‘blood Symptoms may ast aweek or longer _EEREIMEARFIE and Retter's syndrome can develop following infection with a mumber of enteric pathogens incudirg Shigella ‘Salmonella, Campylobacter ané Yersina ‘A 40-year-old woman presents with dysphagia and weight loss for three months as well as vomiting which she describes as containing recognisable pieces of food. She reports that the symptoms are progressive and that itis predominantly solid food which she cannot tolerate, but on further questioning she reports she has had trouble with liquids too. ACKR shows what looks like a fluid level behind the right heart border. Which is the most likely diagnosis? lease ct 1 option) Achalasia Intususseption Multiple sclerosis, © Oesophageal adenocarcinoma Pharyngeal pouch er, ooo I a -—---] i at Pranmaselouch Explanation aha the nity ofthe loner oesophageal sphincterto relax. cusng funciona stctuingof he oesophagus Paves siay complanerajsphag to etn so4s aig hough he eres sry mareccmmon. “cnetacay may cemneate ne presence o fui leelbeind ne eat Baran swat may snows clscalbbeak sppearance, and eatment pinay encosopir surge wn eesopagen ataton or sua meta. -ramygeatpouch an option mts cena however ptn’s classe tpet a gusing sensation inter oatandalso compainof aoa SengeaANClasseatyasnered won sgrcat weet ss anc pogressivedysoha ‘one nouseipect cher stone smptams FNS wate neundewingdagnoss ‘ntusussepstionsapredominany tenn hiker where one ae of bovelsimaginted bythe cer producingbleay ‘rescurantjely 008. sot ane engi Which of the following drugs is a recognised cause of pancreatitis? (Piease select 7 optic > Alencronie acid © Amiodarone © Amitriptyline © Atenolol Azathioprine ——E— Aleniitonic acid o Amitriptyline Atenoial oe) Explanation ‘The correct answeris azathioprine ‘The remaining listed options are not known to cause pancreatitis. ‘Shetetaingtuctenfr nt pine arch ben vi netted ern She ar oon apctonzumpienter ther esas thn fer spt, loesignions moon met asses in on ony spate opm oa, ow some fsa Hoven Neen sith ay apr? | Orne © cues enone © Mute enszcrne nore 0 | Sabena aw ae Cot nase Casings some Mekocone ccs ae) Boplanation “TheCRPisnotraed mating isgnossof robs. ‘Te qvocen cersI0U5 Ras erste espa lengthy eaten wR ‘Sra bvel phoma = suggest rating fe inesne lune stn nptnurbilcalpainmade ware yeti, ‘th wegtows vortng andoseael eect ebsttion Sol owe mga asgneeedby conte adorn ancinestinadopy. “Thermest tel gnosis heres MEN: hey MENTS We snsrome util Us mates aero of ZSERIES ly, dt stoma ypemasaemamayDete case ctthe azote “Thereisasohyperceisas en of thepsthyridhypelsiinccatve of tscanstion Theremaynotrecessniy besa isn spore asesmabeuy18cofnewcanes “Thsinfeity woul ft aprlctnoma Which of the following dermatological conditions is associated with oesophageal carcinoma? (Plea: lect 1 option) © Acanthosis nigricans ° Ichthyosis © Necrolytic migratory erythema ° Tylosis © Vasculitis ee w | ltt Se es . bits ITD Explanation _Acaiiosigsca is associated with gastric adenocarcinoma efit is associated with lymphoma, Glucagonoma is associated with necrolytic miaratory erythema Malignaney-associated vasculitis is associated with haematological rather than solid malignancies. A 53-year-old woman with rheumatoid arthritis was referred with iron deficiency anaemia. Endoscopy revealed several superficial antral erosions ith small bowel biopsy showing mild villous blunting, apopotic bodies, occasional eosinophils and mild increase in chronic inflammatory cells. Colonoscopy was reported as normal. Whatis the most likely cause of these findings? Coeliac disease Q Crohn’sdisease Non-steroidal anti-inflammatory drug therapy ‘Small bowel lymphoma Whipple's disease [5 comeainne Sins | | Nene antintannatey duet SE © Smallbovelmshome ESE | mt toee Explanation ‘This salient features inthis patient's ease revelve around the fact that she has BUR Atl thence the requirement for NSAIDs), the fonidaficighigy anaemia and the superficial ulceration on endoscopy with features indicative of inflammation due tothe chronic NSAID use. RIBS EER associated wth vilous atrophy and lymphocyte ifitration. There is no suggestion on the biopsy of |ymphocyteinfitration which argues against ymphoms or coeliac, Which of the following is correct regarding reflux of gastric contents into the oesophagus? Ple: ° ° ption) Can be excluded by anormal appearance at endoscopy Can be improved by Helicobacter pylori eradication Isacause of asthma Is neutralised by bicarbonate secreted by the oesophageal mucosa Occurs during transient relaxation of the lower oesophageal sphincter ‘Please select 1 option) Canbe sxdea ba nsal appearance atendacopy Canbeimprevedy Helicobacter pyeriradeaton Inaeause ofasthina Is neutreedy bicarbonate screed the oesophagal muesss © Oveusumatvancentretaationettnelower oesoonagea sonncter EE Explanation Diagrosisisbased predominantly on istry witha proportion of paiats wth efx cicease having normal endoscopy. 1 pyfoneraictonis nested in ang term healing feast and duodenal ulceration, but nat refx disease, hist tis tue the cesophageussecsets bcerbonate he statement hereis nt tue Ii faty weak defence and isnot able ‘toners sas contens which eux u the osophesus, More fective Brunner’ lon which secrete alkane ‘mucus te foundin te duederum, “The ink betwean asthma ana ee agSURRHRERR a complex one but recent systemic reviewindetes that “theresa slginesntasociaton butter isa lack of data onthe decton of aul. is therefore not possible to cence ‘that GORD sacause fasta Youare asked to review the blood results of an 18-year-old woman who is known to have anerexia nervosa, She is under close review by the dietician who has asked you to ensure electrolytes are checked daily Which of the following is a feature of the potentially life-threatening complication this lady is at risk of developing? 1 opti © Hypercalcemia Hyperkalaemia © Hypermagnesaemia © Hyperphesphataemia © Hypophosphataemia miso ‘yeaa sl eens AT omorene SI planation Tere repent a ceoliatons wich can rest rmreeesng patents whohsichidperios of tanaton incising these wh anoecareosa Hipepesthateriaiss ey fete ofeeedng sre. elec syndromes dada te cnc complzston which aise asa consequence idan eco shits uring therutional upper of manourshed patents. elec ndtome como: + Hypopherptamia saloon 1 Hypamapresiema ‘+ Datcenses in aris erupt, tNarin and vaceminea nt + huis overoacovtncadems. Trefusretnton mayconibuteto case fahre Brodin KP ast ce stooleuture hestxtay Nomalana parasites seen sou, sour 88 me/t Negative Ssmalliht pleural fusion noted Which othe folowing investigations would be of most diagnostic value? (Pease select option) © Hepatitis serology ‘spmoicescony (0 Stool microscopy for ov, cysts. and parasites © Typhoid serology ° Usaeound sean ofthe abdomen 45-108) asp 10) ssetoPaxstn He 2 Hisabdomen was © MopatitsEsaroey Slenalcscopy © Wahoos © atoms scanofte abdomen REE Explanation Thepresennon ot onsite wh nents inet, “yo serology ureabi ‘Tecere elagnosslsmany pyogenic oramoedi erases Pyopenc abscesses presen wih sngng tes, netrphliaand igh efsmmatan mater ihtsded pal ion tne cormen ad og cures se oan pti The reennten focei SERSEES AL ry sear Mastptrtce not hase owe spp ty rae smoebe sts ae feundin stalin ass han Ser power ces TALA. Sarclogythe marta fiero Uscundscanwouconfem most maceuteazectolageliverabeces and cau uie a dagosteaeptor Smatlesons are bes demonstatadty Cor, What is the approximate incidence of forming pigment gallstones in patients with sickle call cisease? O 30% QO 50% 20% 90% 30% 80% Explanation Patients wth haemolytic sucha scl cel ease nd neta Sata ve an eporoxmately SOS inedence forming gent gastos. Black pigment galstones a common in sickle cel disease al ar® dus to an increase in bun excretion Ther smal size axes © | Mesenteric ischeernia © Non-steroidslanttinfammetoy ves QEESEmeemmerey C Ulcerative colitis Explanation ‘She has'melancsis cal asa result of prolonged iaxatve use. ‘Often the Bowel mucosa looks dark and stained’ during eolonoseony. ‘She may be predisposed to GORHRSHON due to immobility from her arthritis and/or use of constipating pain Killers ‘A 64-year-old man presents to the hospital with bleeding, He has a heavy dependence on alcohol. On examination, he has petechiae and bleeding gums. What is the likely vitamin deficiency? (Please select 1 option) Oo & OB Q)/'c OlE ee ae planation cote hnemsotrptonealtngr mute oben ics Vinee ray. esti pear raknortane ad badge ‘arin ao nom senate egees ik 1st aretesmtesedy eat, “ecaewch sneer uncfcligen fat wen marae tein han ‘amen se ore ese ener 30 fase abspbn Fen Cri semecelcmay chance incr Ey omitre enschede “salem Tan popes todyanen apa nema Eetyees pleas hee Pr ‘Soweto mass pica ra pet woubaigan DOMES Inadion he ante cnurclhemartesfarepderlhacecpeandctalpeacantestn inten stapes cedera aces cxduc eres. a sees eno + rermamnecaeutin nantes enepalnany + vam Bo pnocotalannc eu nse cement espace + Mion Keanentinateigiateecsad vented case nro eee + Viam Eefcenycnreitinyepaies reuopee neredee eis Which of the following statements regarding colon cancer is correct? (Please select 1 option) © In familial cases the inheritance pattern is typically autosomal recessive © In femiial polyposis coli, the increased cancer riskis due to inheritance of a mutated tumour suppressor gene O Innon-fernilial cases, gene mutations in the cancerous cells are unusual © Itisa characteristic feature of the Peutz-Jegher syndrome © ftoccurs most commonly in the ascending colon Info ass theinhertonce ptm teal avtosoma rcesie ‘nfo aos oo, the neresea cancernekie dus to inettance ofa mutatadturursusreetor gene ‘nnn fami cates, gone mutationsin the cancerous as re unusual ‘ta characertifesture ofthe Pts ephersyndome © occu most commany inthesecsndngclon EER Explanation Bot familial pokposs col and Cashes ndtomeareautosoal dominant. Anal deletion of putative tumeur suppressor gene located 5921-422 eli atenomsteu polyposis APs an autosomal dominant dover causing arene acenaratouspohps ofthe colon and ear onset coretl ance ‘ouanerate and qualtaive ater n gene expression accurate BTRAIEAREE ces These ncuaeaeratonsct po-oncogene apressian and hvemosomal abnomalties letionat 17p and 18a ae sen in 70% elorectal “arcnomse PeutJagnarssynaromils ominanty inher pigmentation ski nd mucous membranes nhamaramatous pohpsin ‘the stomach and gents. The pops ony rey undergo magna change “he rectum ae sod exon are the commonest tes tthe ascending eon A.29-year-old male presents with symptoms of severe gastro-oesophageal reflux despite prolongedcourse of PPI therapy Which one of the followingis most useful in assessing the role of surgery? (Please select 1 option) © Cardiac sphincter manometry Gastric emptying study Intragastric pH monitoring off therapy ) Gesophageal motility and pH study Water soluble contrast swallow study, eS Caster sity cxepngalmey anc ucy I © Witenes my Explanation anrscoifiietaton sth treatment of hoe fr tens th GORD refactor neat of. proton pur “Thepater tohavehadan endoscopy hn thes mnths pro auger to mld uraurpactedpthlgy sch ‘ncesphape tans sy indestesto eou 2prnay motor derderor example eli erode when suspeete ana leout pers which my su npstpertne shag afersomefrmsotfundcoeston. Nice giearcestnehatotoning'sophagen manors and aru 24 hour escphage ‘quant tran este elon bebsanefucepatdesand the persone symp) “Theltervetantonmay to ecudeceazphags mes doce: suchas chs and aevere oneal ypomoniy sunt sesodemah cesopagus tore consonng anes A.42-year-old female with ulcerative colitis is found to have anti-smooth muscle antibodies. Which is the most appropriate next test for this patient? (Please select 1 option) © Abdominal ultrasound © Colonoscopy © Fullblood count © Liver biopsy Oo Liver function tests © ‘Abdominal utrasound Colonoscopy Full blood count Liver Biopsy © Lverfunction ests EES Explanation “The most appropriate investigation for this womans LFTs to begin with to assess f there are any features of autoimmune hepatitis, such as, ised bilirubin, aspartate aminotransferase (AST) alanine aminotransferase (ALT) and alkaline phosphatase. If this isthe case then liver biopsy may be requived or further diagnostic imaging, Autoimenune hepatitis is often seen in individuals with other autoimmune disorders such as ulcerative colitis, Which of the following hormones stimulates contraction of the gallbladder? ° ° Cholecystokinin Gastrin Secretin Somatostatin Vasoactive intestinal peptide (VIP) coeccnn Ty ccxstin © Seeretin Somatostatin © Veorciventetns pence) Ey Explanation ‘castin leads tothe release ofgavicaci, ‘ecretin stimulates te rolease of pancreatic uid and bicarbonate ‘Somstostatin inhibits gastrointestinal endocrine secretion Vasoactwve intestinal peptide (VIP) functions as a vasodstor and also regulates smooth muscle actwity epithe cell secretion ‘and gastrointestinal blood flow. ‘835-year-old woman with Crohn's disease has had several episodes of relapse which are refractory to oral and rectal steroids, {and to azathioprine. She presents to hospital with severe bloody discrhoes of three days curation, associated with abdominal pain and perianal ulceration On examination she looks pale and lethargic and baseline observations reveal BP 60/60, HR 120. Allher other observations are normal She i admitted, given intravenous fuids, and following further unsuccessful attempts at mecical management, undergoes colectomy and resection ofa significant amount of ium, with end leastomy formation You review her on the ward three days later when she complains of severe watery larchoea whenever she attempts to eat Which ofthe followingis the most likely diagnosis? lease select 1 option) © Hypocalcaemia © Ongoing active Crohn's disease © Shorbowel syndrome 2 Vitamin D detiiency © Zinedeticieney ypoclcema Onapingacive ots dsrase eo one amino eekiny Se ET Explanation Shee ccs when tee he been euactn of asgnican portion ofthe salintasine suing risborpionofrutents Management shoud ely acs en eplacement tae nd latches Catrerypeecttion shoud be managed wth eton pp inboard peramide x cde canbeusedo sew asic anabowe tans Teatent souk inealybe kept youth reduethe osmate oad Specs teams, nung dts sould hente costed ‘eaicngine most appro tandngta ie The Bowalttan aca ter ein oxeoperate prod snd eeecng, should bein sow thes a he ptt ingame ral, ‘Astnerenasbeenasgtean nowairesetin, whines) Been ae ensre ening nsiceessmedks ‘management une the Peta tie would have the patent wih bowelctey act Groh ea ypocsesena, Mimi en) ana Bate canal est Tom smullowelsynctome, bt tese eager ttm Corlcaionsandao ot pcaly present soacutey androt wenths neice A 28-year-old male presents with a four-day history of profuse bloody diarthoea after retuming from a holiday in the Far East. Which of the following regarding his illness is true? Piease select 1 option) © Anegative amoebic fuorescent antibody test excludes a diagnosis of acute amoebic dysenty ° Cysts to £ histolyticain the stools are only seen in acute amoebic dysenty © Choleraisa likely diagnosis © Giardiasis isa likely diagnosis O Shigeliosisis alixely diagnosis Anegative amoebic fluorescent antibody test exciudes a diagnosis dysentry histolyacain the Cholera isa likely diagnosis Giardiasis is a Ikely diagnosis Shigellosisisa likely diagnosis ESE Key Learning Points Gastroenterology, Infectious Diseases © Shigeliaisa cause of profise bloody diarrhoea, Explanation ‘Shigellosis is a possible cause of profuse bloody diarthoea as ghiblets and Biatdlasié are associated with watery diarrhoea ‘Trophozoites and cysts are seen in acute amoebic dysentery, however, cysts may also be excreted in asymptomatic cartier states. A 45-year-old gentleman presents with dyspepsia of fve months duration and loss of weight. Examination reveals mild pallor and slight epigastric tenderness Gastroscopy reveals 5 mm posterior ulcer in the first part of the duodenum and 2 cm mass on the lesser curve of the stomach. Biopsy of the mass reveals mucosa-associated lymphoid tumour confined to gastrie mucosa He has tested positive for H pylariinfection. Which of the following treatment options will be appropriate for him? Please select 1 option © Chemotherapy © | Hpylorieradication © Proton pump inhiitor © Radiotherapy ° ‘Surgery © Chemetherany: pence TD © | Proton pump inhibitor © keccherey Een 2 [see Explanation Lymphomas restricted tothe gastrie mucosa usually appear when H pyletis eradicated ‘These lesions are less likely to respond to H. pylorieradication alone if they extend beyond the gastric mucosa. Chematherapy ‘or surgical excision may then be indicated. Duodenal leer wil also disappear with Ht pyforieradication, ‘A.28-year-old man with HIV presents with a five-day history of feeling unwell. He isa heavy smoker. A chest radiograph snowed right upper lobe consolidation. His CD4 count was 468 cells/mm?. HIV RNA level was 90,578 copies/ml. He is not on any antiretroviral treatment. Which of the following is the most likely diagnosis? select 1 option) Bronchial carcinoma ) Invasive pulmonary aspergillosis Pulmonary tuberculosis ) Pneumocystis jiroveci pneumonia PCP) Streptococcal pneumonia eee aeRO eae eae 2 mmgstiomspennne ET Seccipeeces Ee Explanation “nisi picalet commun acquired anaumeni Onesnould thnk of cmmen nection rtherthen ay operas "fectonsin patents wh good CD counts. North 400 cals? sntimrunocempromiced) Itisnot atypical histor forbronhis carcinomas the isry wes short ns aig Invasive pulmonary ASE unin patient wth 00d D4 coun _mnanaP RUE ialy causes cavtztng sions inapaene wth agoodCO4 count cP eummary cceursn patints wth CD4 count eee than 290 seem? anc ene dog shows steal ‘tom hes wthout ny eFuxion or nghadenopt “The hetoryis to short fr tubers and apart rm HV here ae any ther ik factor fortis. pic chnges wer net piealof Which one of the following is an oncogenic virus? (Piease select 1 option) Hepatitis A © Human papilloma virus 6 (HPV 6) © Human papilloma virus 11 (HPV 11) ° Human papilloma virus 16 (HPV 16) © Varicella zoster virus (VZV) Hepatitis A Human papilora vitus 6 HPV 6) 3 Haran ppilom ins 1 V1) |e Human papillomavirus 16H 16) @EED |e Varela zostervinis O21 Explanation HPV 16 s oncogenic ané causes squamous cell carcinomas in the oa cally, cervix anusand pens, BERLE i not an oncogenic virus, asit does not eause chronic infection of cancer. HPV 6 and 11 typically cause the majority of benign warts, ‘VZV causes chicken pox and herpes zoster. A 48-year-old African man with HIV was prescribed a combination of antiretroviral therapy. He developed increased ciffuse pigmentation of the nails in both hands and toes. Which of the following is the most likely cause of the increased pigmentation of the nails? Please o © Lamivudine Nevirapine © | Tenofovir © Zidovudine | Htavienz Explanation “idowudine causes increased pigmentation othe raisin black patients Efawrene causes central nenous system toxicity not hyperpigmentation. Lamivucne does not normally cause hyperpiamentation of nals but itcan occasionally cause hyperpiamentation ofthe skinin biackpeople. [Nevirapine doesnot cause hyperpigmentation of skin but can cause acute hepatitis and skinrash. ‘Tenefowir can cause pronimal tubular damage hence Fanconiike syndrome. Which one of the following drugs is associated with hypersensitivity reactions? (Please select 1 option) © Atazanavir © Lamivudine © Nevirapine © | Tenofovir > Zidovudine ‘acon | Lacing one | reste ns ) Explanation "Newrapin can cause acute hepatitis and skin rash a part afhpersenstive reaction espacial when he CD4 counts over -2s0cel/iin ween anid ver 400 cells/ml inmien Nevrapine should note pescrived in those condions. ‘Marana causes hypertilubinaemia and rarely renal stones. Lamividine doesnot cause hypersensitty reaction. Tenofosr causes pronimal tubular damage. down caus bone marrow suppression Which of the following is a sign ofimmunodeficiency in the mouth? O Gingivitis © Herpes labialis © Leucoplakia © Orathairy leucoplakia © Oralwart Herpes bias = concn aoe) Explanation ‘ral hair leucoplakiais a sign of immunodeficiency tis due to reactivation of Epstein Bar viusinfection SBS snot sign of mmunodericiency. Leucoplakiaisnota sign ofimmunodeticiency but itis a precancerous lesion ‘Herpes labialis is due to herpes simplex infection, which causes cold sores'inimmunacompetent patients and chtovic herpes Jabias in immunocornpromised patients. (ra warts can occur in heathy people. They ae due to HPV infection usualy due to benign types, 6 and 11 Which one of the following cutaneous lesions is associated with HIV infection? +t 1 option) © Leucoplakia © | Lichen planus © Lichen scierosus © | Plasma cell balanitis. © Psoriasis | © Leucopiaki [© chan pans Lichen setorasus © Plasma cellbalanitis oes Explanation ‘Leucoplakia is a pre-cancerous lesion andis a sign of immunodeficiency in an HIV-positive patient. Lichen planus, chen scleresus and plasme cell balanitis are not associated with HIVinfection. [pre-existing psoriasis lares up for no apparent reason or middle-aged people develop psoriasis for the fst time, one should ‘exclude underlying HIV infection in those patients thougn tis nota typical ign of primary HiVinfection. 30-year-old Caucasian man with symptomatic HIV disease developed multiple, painless, umbilicated papular lesions on his face. Which of the following is the most likely cause of his skin lesions? (Please s: © Cytomegalovirus (CMV) © Epstein Barr virus (EBV) © Human herpesvirus HEM 8 © Human papilloma virus (HPV 16) © Poxvirus 2 Cytomegalovirus (cM Epstein Bar vrustEB) © Human nenesvinu h9 © © Human poptoma ius He 16) ERAN roves Explanation Mutile painless umbilcated papular lesions are typical of mluSCUim COntaBiGSUh ond are caused by pox virus. ‘CMY does not cause painless papular lesions BV causes Burt’ mphoma, non-Hodgkin's imphoma, nd primary bain ymphomas. HEV 6 is strongly associated with Kaposis sarcoma, HPV 16 associated with squamous cell carcinomasin cervix penis, anus and ora cavity. A51-year-oid homosexual, Caucasian, HIV positive man developed multipie violaceous painless lesions on his trunk. Which one of the following is the most likely cause of his skin lesions? Please select 1 option) © Cytomegalovirus (cmv) Human herpes virus 8 (HHV8) © Human herpes virus 10 (HHV 10) © Human papilloma virus 16 (HPV 16) © Poxvirus > Cytomegalovirus cMVy Humanhepesviuso(Hive) GSES Eee Human herpes rus 10 OHH 10) Human papilloma virus 16 (HPV 16) © cons SET Explanation "Multiple violaceous painless lesions ae typical of Kapost's sarcoma in Caucasians. This is associated with HV 8 (CMV and HHY 10 do nat cause multiple ilaceous painless lesions. HPV 16 isan oncogenic virus which causes squamous cel carcinomas. Pox virus causes malluscurn contagiosum, ‘A.19-year-old man presents to the Emergency Department with tea-coloured urine two weeks after the onset of an episode of, streptococcal tonsilltis for which he received a course of Penicilin V. He has also noticed puffiness around his eyes which has, developed over the past 24 hours. There is no past medical history of note and he takes no regular medications. BP is 148/85, pulse is 60 and regular. You confirm periorbital oedema on examination. Urine dip which is positive for both blood and protein Serum creatinine is 85 micromol/I, (60-110). Which of the following is the most likely diagnosis? lease select 1 option! O Acute interstitial nephritis © IgAnephropathy Mernbranous nephropathy ° © Post streptocaccal glomerulonephritis Minimal change disease pee rerstianep his JeAnesrcpany Memornous nephopaty © Momaicence ssc: ET Poststeptococcal gomewionestts EIS Explanation “The ansnersposeseptococa SNMGREGIS Camus poet MREUNRRRIED. The me period 2 weeks ster the acute infections clase asisthe presentation with discoloured urine ane perioral oedema Hypertensonis ‘seenine0% or moreofcass,athough tisusualy tansientand resehes onnrmalsaon of lasma volume: Teatmentis ‘imal supportive, atinough some aduts may be eft wth sme longterm renalimpaiment. RRA a2 result of pein starts 2-0 days afer hrapy srt and iaseocatd witha feverand ‘macuopopuar ras as wel ashacmatura would detnitely be apossbtyin tis ease, however you would pect more ‘Sgncartrenalimpaiment than seen hee ESMopah presents mare quel after respirator tactnfction (24 48 rout) usualy wt naematurs Membranous disease peers with nephrotic yntromeand predominant poten ririmal change deco f more ely to presen with repesied episodes of neproticrange atin in chishoodand | ‘teenage years A36-yeai-old male who has presented with chest pain, has a PSA of 45 ng/ml (normal <4) Which of the following statements is correct with respect to this patient's management? ase select 1 option) © Anelevated PSA is a definitive test for prostate cancer © High selenium intake is related to prostate cancer © Prostate canceris more aggressive with increasing age © Prostate cancer is typically squamous call carcinoma © Themost commonly used pathological grading systemis the Gleason score © Anelevated PSAs derive test for rotate cancer ignselanum karat to posite cancer Prostate cancers gps wh renin ge Prostate canceristyicly squamous calles © Themoscommorivuedpotebsicaaadngsrtem'stheGeeson coe EE Explanation Prosatespecifesnigen PSA0maybesesstedin: * prostatas * benim mesichvaraia ond + prostatecance: ‘Asal prostate more sapressbein younaermen, PSRs on adenocarcinoma. ‘The Gleason sores recmened bythe American Clegecf Fatologsts. The most wel dierentates wumeurshavea leson score. andine most poy read a aso sore 10 nate of animal fs inane SRS las ow ina of seein, _AzD,yofd student comes the dn comelaning of sur andminr ro aIngan pan Hes Ae roid a ‘oneuennstonhstpatue 37S stun smal anstnderanyoucen ese cpu ‘teoarge rine wet aa, Insteonstow aonb 99 ass ss rearnora spo seunseoun etme os0 seumpans smn as inn chore poste © Aetromyon parole tne lady ogee cy ro © mncjne tome say a2 © Metesin sooner ae ° Fenainy sto mes0.0r7 ee eae 1 ce, —_Aaithromycin 1 gas single dose followed by 500mg once daly for wo days es Crafotacn S00 me BD For7 days Minocyetine 100 mgcaiy for 9 days Norflcacin 400 meg daily for7 ays Penciinysoomsso%e 7 oo SRE Explanation “The answers atiromyeint gas single cose alowed by S00mg once daly er two day Ina student population whese compliance may well De problem shine single dose of antiotis forthe treatment of Chis the most sensbie option Other options for reatment of Chamydaincude minacyelne although daxycyctine causes less gastrointestinal disturbance. (fioxacin 200 ma BD for seven days is also considered a potenti option accordngto SIGN auitines. He should alco be referred tothe GUM cinic for screening fr other soul wansmitted infections. A67-year-old male presents to his GP with urinary frequency, urgency, nocturia, and poor stream. An enlarged prostate is felt on digital rectal examination. Urine dipstick shows nitrites, leukocytes, protein, and blood. He is prescribed nitrofurantoin. His GP wuld like to check his PSA When should his PSA be checked? ct 1 option) © Checkin three days © Check inoneweek Check in twoweeks Check on this occasion No need to check PSA Ccheckintvee dye Chechnonenect connec IY Neneedto creck PS Explanation 54 concentatonscan ise wthorcblemsof fs poses ater ahetrsation and pricully ine wctinfctions “hetfce tones cided to chack a PSA nents acornmended ats ause mest a bt bs wake ae ‘vest. Pose besrinmina natn conveyer ares and one where the uienesarenot complete cet ‘Youwould epect some pestatcsyretors in paints wih past eacnama but mast thas willactualyhave BPH rather than carcinoma Mary men wilomerequestngaPSAanditcan be useful hot patents witha fy ston Youn sere, Inpracice wou potaly be wrt checking ara presenting witha UL. as uch agnosis ay tein ths ge ‘up I simporare tallow teinectontocle fst however Ye reducethe ket afase postive res Which one of the following is an AIDS defining illness? (Pie: © Analcanal warts Extre genital molluscum contagiosum Multidermatarnal shingles Oesophageal candidiasis Oral candidiasis Anal canal wars tra genta mollscum contagiosum cecongelenddess ETD ee) | temas Explanation ‘Any opportunistic infections or opportunistic mabgnancies are AIDS-defining nesses. DDesophageal candiisisisan ADS-detring ness but oral eancidiassis not ‘Anal warts ae not an opportunistic infection. ‘Erogenta!mlscum eontaeasum soso not on opportunist infection but equentlyoceursin symptomatic HV-postive patients Muttidermatorral shingles is notan opportunistic infection but fit occurs ia young person HIVES needs tote eile 31-year-old African man presented with a history of fever, night sweats, shortness of breath, and weight loss for two months. His chest radiograph showed a moderately severe, left pleural effusion only. He consented to an HIV test which was positive. Which is the most likely cause of pleural effusion? Please select 1 option) © Hodgkin's lymphoma © Pleural tuberculosis © Preumocystis irevecii pneumonia (PCP) © Pulmonary aspergillosis © Pulmonary Kapos's sarcoma ‘(Please select 3 option) Hedges terchoma Preumocrstisroreoneurenis CP) ee) Pulmonary Kapono Explanation eur berculosssthe ost ely couse nan HV poste Arcanman wha twementhhiston oT weg es Hs pew ‘one det ple tubercuostraplnonry bere Hedgusymphomacan eas BURT deo pleat moherant butt isaten assocatadwih meastnsimass HE ‘hestraiogzoh showed ery peur eusn. Ner dak mohams ret Hock rrphera}isconmenly assorted wanes patents PoP coe ot ave pours piel causes stra rtuarhadon rr thea without ny iar ymgh neds ‘niugerenterleualemuson, umn REESE stows attra sens butions a type cause pleural eMun, Pumanary Kaestsarcomacan cause GLE by olin ne pleura, But ten causes coarse ep nla A 35-year-old, HIV-positive, African woman presented with weakness of both legs and double incontinence. CSF showed increased protein and neutrophils with normal glucose Which of the followingis the most likely cause of her weakness? Toption) © CMVpolyradiculomyelopathy ) Guillain-Barré syndrome © Herpesvirus encephalitis, © HlV encephalopathy © Toxoplasme encephalitis © CMV poladicuonyelopathy EEE Guiai-baré syncrome a © Hivencephatopatty ee) Explanation “Symptoms ate suggestive of palyraciculomylopathyleskness ot legs with involvement of sphincters) Increased neutrophils are foundin CMW polyadiculomyelopathy but notin Guilan-Baré syncrome. HIV encephalopathy usually causes confusion and memory oss It does notinvalve sphincters, Guillain-Barré syndrome causes pobyradiculopathy, explaining all her symptoms, but with normal call counts and aise protein inthecsr. Herpes simple Sep causes fever, eadsche, confusion, and deteriorating level of consciousness. A.34-year-old Thal lady presented with a left hemiparesis of two weeks duration. HIV antibody test was positive. CT scan of the head showed multiple ring-enhanced lesions. Which of the following is the most likely cause of her weakness? (Please select ©. Amoabic brain abscesses O Cerebral toxoplasmosis © Herpes simplex encepnaitis ©. Primary brain lymphoma © Progressive multifocal leucoencephalopathy ) Amosbickrain abscesses coos TT Heroes snes encepats Stans Se Progressive multfacallaucoancephaloathy Explanation Cerebral bolas the most aly ice Ntplanng-ennancesesions are commen seen in ptt th cerebral HERBEER, though slaying enhanced lesions are seen in 25% of patients on CT scan, MR scans more Senstvein dentin smallesions than CT scan, rmosbi ran abscesses are not the mostikelycauceinthispatint. "Muti ing-enhanced lesions ae not seen patents wth eipes simplex encenhatis Primary brain phoma causes significant mass fect wth sutounding oedema Progressive mulfocal eucoencephalopsty causes mutoca white mater lesions winout en mas eet or surounding oseams ‘A.60-year-old Asian man who has lived in the United Kingdom for the past 15 years presents with painless haematuria. He is a smoker of 10 cigarettes per day. Investigations reveal a haemoglobin of 110 g/L (120-1 60), urinalysis shows ++ blood and PA chest x ray shows small flecks of white opacifications in the upper lobe of the left lung, Which of the following is the most likely diagnosis? select 1 option) © Bladder carcinoma © Glomerular disease © Prostatic carcinoma Renal calcul ° Tuberculosis ere Glomenuiardisease Prostatic carcinoma Reral calcul recess TT Explanation ‘The most key diagnosis in this midele-aged maleis carcinoma ofthe blader as suggested by haematuria and anaemia, ‘The history of smoking sa isk factor for bladder cancer. ‘The gentleman does not have any of the festures of active TB suchas fever, ight sweats or weightloss, which we vcult ‘expect Phe had renal TB. The opacificationsin the lung are consistent with previous primary TB. Having sais that, renal TBean present without systemic symptoms but Bada fs more common, A32.year-old African woman with HIV presents with a two-week history of greenish, frothy, itchy vaginal discharge. What is the most ikely catise of her discharge? (Please select 1 option) © Candia albicans infection © Chlamydiainfection © Foreign body © Gonortheea infection © Trichomonas vaginalisinfection © Candide aticansintection CChiamyataintection © Foreign body © coverecsnecton ER © rahomenasvegnaisintecton (TS Explanation Trichomonas vaginalis causes itchy, frothy, greenish vaginal discharge. Candia atbicans causes a white, urd, ty veginal discharge Chlamydia and gonorthoea do not cause itchy, frothy, vaginal discharge and both can be asymptomatic. Foreign body causes foul smeling vaginal discharge A 34-year-old homosexual Caucasian man developed jaundice two months after taking @ combination of antiretroviral drugs. He admitted that he had had several episodes of unprotected sex with several casual male partners. His liver function showed taised bilirubin with normal transaminases and alkaline phosphatase. Which of the following is the most likely cause of his jaundice? Please select onl O Acute hepatitis 8 © Alcoholic hepatitis, Atazanavir © | Efavirenz 3° Nevirapine > | eines Asha hepa ooo) favrens Nevcone CLAY Explanation [Atazanaviceauses hyperliubinaemia with normal transaminases ane alaline phosphatase (mimicking Gibert's syndrome). ‘Acute hepatis Bis unlikely with normal rsaminases and alkaline phosphatase ‘Ald to moderate sein tasarninases and alkaline phosphetase occurs in aleaholic hepatitis “Efavcen2 can cause acute hepaitis wth raed loves of transaminases. Nevrapine causes acute hepatitis where transaminases ae raised several-fold ‘A 36-year-old Caucasian woman was successfully treated for Pneumocystis irovecii pneurnonia (PCP). ‘She was re-admitted with acute breathlessness with left-sided chest pain ten days after her discharge from the hospital Examination revealed that she was hypoxic and found to have diminished breath sounds on the left side of chest. Whatis the most likely cause of her recent admission? (Please select 1 option Acute myocardialinfarction Acute pericarditis ‘Acute pulmonary embolism. © Leftlobar pneumonia © Pneumothorax SSE TEP One: © aute mypcardiatitatetion Acute pericarditis © Acute pulmonary embolism © Leftover pneumonia Preumothorax ES Explanation PAGAL is wel known complication of PCP An acute history of chest pain with breathlessness and diminished breath soundsis typical of pneumothorax. Diminished breath sounds are nota feature of acute myocardial infarction or acute pericarditis Acute BORAEZEMABOISE should be considered de to her recent admission but diminished breath sounds ae not feature “There arene signe of consolidation to canside lobar pneumaria Which one of the following antiretrovirals is likely to cause Increased pigmentation of the skin in a black African patient? iease select 1 option) OQ Didanosine > Efavirenz © Emtricitabine Nevirapine Stevudine © didanosine Efavirenz cece TS Nevirapine © seve SR Explanation Emtricitabine causes hyperpigmentation of the skin, inctuding palmar creases, n 8% ofblackppatients Didanosine anc stavudine cause mitocheneril toxicity, hence periaheral neuropsthy, pancreatitis anc hyperlactataemia Efavirene causes CNS tonicity. Nevirapine causes acute hepatitis and skin rash. A 47-year-old Portuguese former intravenous drug abuser presented with a two-week history of right hemiparesis. He was found to have hepatitis B and C infection. His absolute lymphocyte count was 0.6 *10°/L CT of the head showed multiple ring-enhanced lesions Which of the following would be your next best course of action? Manage him conservatively with physiotherapy © Refer him toa neurosurgeon for urgent brain biopsy Refer him toa stroke specialist © Requestan HIV antibody test Start thrombolysis treatment ee eee ee Manage him consarativaly ith physiesherapy Refer him to2 neurosurgeon for urgent brain biopsy > Referhim toa stoke specialct © Requestantivenddoaytest EES 5 Storttrombolsis treatment Explanation “Tis man as aready infected with two blood bone vruss SRR and O, His absolute ymphocyte count was low. CT an shomes mutple rng enhanced sions, which wee suagestve of erebvaltoxoplasmesis. “Therefore, tesng HV is te next best course of action, Fncing mutipering-enhanced lesions on CT scan needs further investigations Managing conservatively with physiotherapy isnot an appropiate course of action, (CT scan‘snot npical of bain rumour hance refering him forurgent brain biopsy is nat the best course faction “Thrombolysis weatment shoul nt be started asthe CT scan was not typical of schagmic stroke A 46-year-old homosexual HIV positive man presents with a two-week history of weakness of his right arm and eg. Examination reveals right hemiparesis and left cerebellar signs. CT scan shows white matter lesions in the left cerebellar region and left temporoparietal area There is no midline shift or surrounding oedema Which ane of the following is most likely to be found in his cerebrospinal fluid (CSP? (O Positive cytomegalovirus (CMV) PCR Positive Epstein-Barr virus (EBV) PCR ‘© - Positive herpes simplex virus (HSV) PCR Positive human herpes virus (HHV) 8 PCR Positives PCR Postve ctomegsioveusfemO PCR Postve Este Bar vius €BVIPCR Postvehespes simplex vs 450) POR —— | reser ee Explanation Mutfocallesinsin tet cerebelar and tenporopaital white mater areas without any mass fect orurounding oedema ae "ost key tobe due o progressive mutoalleucoeneephalopsny (PML, |e virus causes PMLinimmunocompromised paints especialy when the CD4 count isbelow 109 cali/mm? ‘CMV poymeras chain eactionPCRI may be foundin CMV encepnalts itis cnicaly not atypical feature of CMY encephalitis Poste EBV PCRndiestes primary brain hmmphome where CT scanften shows sgricant mass eect with surcunding ‘secera, HSV FH may be founelin HSV BSR which commonly affects temporal lobes in patients ith goed CDA count HEV BCR s usually assocatedwth Kaposs sarcoma,

You might also like